Anda di halaman 1dari 99

re" i.E +H.WsdF,l .

f,nrnpl*tm exr:*i r:**?milm#, #is*ussr*n *f th* f*rn'rt *lg$ and


Pll,:le iP!'*$ '

ffri*r 5** s,*iu*,J ,ir]# lu6:p:l*rx*nfffiry prsblems


il[.s,{*friit*.xhi*s r}*# {:*ftv*r$r*n fqx***r$ in th* Appendix
Wrtf] i*,*tlx f*.:r r**sy {:***ss t* t*pl*S in fhe b*mk
i ne;: r.: *j <*g t*il * trr
I
j Lr * rg,'# e*;qr* ry:$ rrffi li*r'r q *estlqns

www.gertcrev.com
TABLE OF CONTENTS
I

To my motherlluminada,
my utife Imelda,
and-our Children Kim Deunice,
KenDainiel
and Karla Denise
TABLE OF CONTENTS TABLE OF CONTENTS ilt

CHAPTER 3
Total Hydrostatic Force on Surfaces............... ........T9
Total Hydrostatic Force on Plane Surface ................79
Properties of Common Geometric Shapes... ........76
Total Hydrostatic Force on Curved Surface...... .......28

Analirsis of Gravity Dams... ................84


Buoyancy. ............:................... 88

Statical Stability of Floating Bodies ........90


Stress on Thin-Wa1led Pressure Vessels ...................96
CylindricalTank. ...,...,....,:.96 CHAPTER 6
. Spherical Shell ...................98 Fluid Flow Measurement.......'... """""277
Wood Stave Pipes......... .....98
SOLVED PROBIEMS..................;................ ..99 to 195 Head lost in Measuring Devices...........'.....:. "'"""''279
SUPPLEMENTARY PROBLEMS ...............,.................. 1 96 to 2OO
Values of H for Various Conditions """""""""'283
Contraction of the Jet...........'.. ,,.."""284
Orifice under Low Heads................. """"""""""285
CHAPTER 4 Venturi Meter ....285
Relative Equilibrium of Liquids............
Rectilinear Translation ...................
...,............20L ffitT"...... """""'287
........ 201 pitot Tube .............................. 288
Horizontal Motion ..........201 Gates """"""""" 290
Inclined Motion...... .,.......202 -
I uDes......... """"291'
Vertical Motion...... ..........203 Eilt"ady Flow (Orifice)..'.........' .........'.."""" """""'294
Rotation ..............202
Volume of Parabo1oid............... .........20s --elassification of Weirs'."...' """""""'297
. Liquid Surface Conditions ................206 Rectangular Weir'...".',.!'.....'.!..;". ""'298
soLvED PROBLEMS................. ..................210 to 240 Contracted Rectangular Weirs "" 301
Triangular Weirs .....i...'..r...... """"""' 301
Trap&oidal Weirs...... ""' 304
Cipolletti Weir ......... """":""""""' 304
suttro weir..."""' """"""' 305

Er
V
iv TABLE OF CONTENTS TABLE OF CONTENTS /-
CHAPTER B ,,',,481,

,,.,.'''.' 500
Flow AtULrrlLl
rIUl/v around \-rlctlulEr Bends........
Channel pslluD........ ...'............'.1,, n1,
...'""""""".n z ,.lnn
to547
soLvEDPRoBLEMS................ . .. tun
CHAPTER ONE
VI TABLE OF CONTENTS FLUID MECHANICS
Properties of Fluids
& HYDRAULICS

CHAPTER 9

'
Force against Fixed Flat Plates ..............5S1
Chapter I
Force against Fixed Curved Vanes ....... S53
Force against Moving Vanes ................. S54 Properties of Fluids
Force Developed on Closed Conduit.... ..................556
FLUID MECHANICS & HYDRAULICS
Drag and,Lift............... ..........557
I:hii Meclunics is a physical science dealing with the action of fluids at rest or
ln motion, and with applications and devices in engineering using fluids.
Water Hammer...................... ..................560 Fluid mechanics can be subdivided into two major areas, fluid stntics) yvhich
clealswith fluids at rest, and fiuid dynamics, concerned with fluids in motion.
SUPPLEMENTARY PROBLEMS .................................. 5 97 to S9B The term hydrodynamics is applied to the flow of liquids or to low-velocity gas
flows where the gas can be considered as being essentially incompressible.

with the application of fluid mechanics to engineering devices


l-lydrarLlics deals

APPENDIX involving liquids, usually water or oil. Hydraulics deals with such problems
as the flow or nuias through pipes or in open channels, the design of storage
Properties of Fluids and Conversion Factors .,....5gg dams, pumps, and water turbines, and with other devices for the control or
Table A - 1: Viscosity and Density of Water at 1 atm..... .........599 use of liquids, such as nozzles, valves, jets, and flowmeters.
Table A - 2: Viscosity and Density of Air at 1 ahn...................600
Tab1e A - 3: Properties of Common Liquids at 1 atm & 20"C..601
Table A - 4: Properties of Common Gases at L atm & 20"C..... 601
TYPES OF FLUID
Table A - 5: Suriace Tension, Vapor Pressure,
Fluids are generally divided into.two categories: ideal fluids and real fluids.
and Sound Speed of Waier ...,..602
Table A - 6: Properties of Standard Atmosphere..................... 603 ldeal fluids
Table A - 7: Conversion Factors from BG to St Units.............. 604 . Assumed to have no viscosity (and hence, no resistance to shear)
Table A - 8: Other Conversion Factors .lncompressible
...................605
r Have uniform velocity when flowing l

. No friction between moving layers of fluid


. No eddy currents or turbulente
INDEX I - IV Reat fluids
. Exhibit infinite viscosities
. Non-uniform velocity distribution when flowing
. ComPressible
. Expeiience friction and turbulence in flow
f

CHAPTER ONE FLUID MECHANTCS CHAPTER ONE


2 Properties of Fluids & HYDRAULICS
FLUID MECHANICS
& HYDRAULICS Properties of Fluids 3
Real fluids are further divided into Nerutonian fluids and. non-Nerctonian where: p = absolute Pressure of gas in Pa
fluids.
R = gas constantJoule / kg-'K
Most fluid problems assume real fluids with Newtonian characteristics for
convenience. This assumption is appropriate for water, air, gases, steam, and R=287J/kg."K
other simple fluids like alcohol, gasolir,e, acid solutions, etc. However, R= 1,716lb-ft/slug-'R
slurries, pastes, gels, suspensions may not behave according to simple fluid T = absolute temperature in
oKelvin
relationships oC + 273
"K=
oR='F+460

Table 1 - 1: Approximate Room-Temperature


Densities of Common Fluids

Fluid p in kg/,ml
Air (STP) 7.29
Air (21'F, a Ltm) 1,.20
Alcohol 790
Ammonia 602
Gasoline 720
Glvcerin 1.,260
Mercury 13,600
Water 1,000
Figure 1 - l: Types offluid

MASS DENSITY, p (RHO) SPECTFTC VOLUME, Vs


The density of a fluid is its niass per unit of r4olume. Specific volume, %, is the volume occupied by'a unit mass of fluid.

p=
massof fluid, M Vr= L Eq.1-3
Eq.1-1 p
I
volume, V

Units:
English: slugs/ft3 Note: pstug, = ptbn/ I
Metric : gram/cm3 UNIT WEIGHT OR SPECIFIC WEIGHT,t
SI : kg/m3 Specific weight or unit weight, y, is the weight of a unit volume of a fluid.

For an ideal gas, its densify can be found from the specific,gas constant and
ideal gas law:

y=pg, Eq.1-5
o=
'RT
P
Eq. 1.2

*!r
T
CHAPTER ONE FLUID MECHANICS FTUID MECHANICS CHAPTER ONE
4 Properties of FIuids & HYDRAULICS & HYDRAULICS Properties of Fluids 5
Units: llro upper plate will adhere to it and will move with the same velocity U while
English lb/ ftt thc fluicl in contact with the fixed plate will have a zero velocity. For small
Metric dyne/cm3 virlues of U and y, the velocity gradient can be assumed to be a shaight line
SI N/m3 or kN/mr rrrrtl Irvaries asA,U andyas:

^AU
toc Of
FU
-v Ay
-0c-
SPECIFIC GRAVITY .U
but
dv
(from the figure)
Specific gravity, s, is a dimensionless ratio of a fluid's density to some ydy
standard reference density. For liquids and solids, the reference density is
water at 4" C (39.2" F). +A = Shearingstress, r

t oc
dv Of T =k-.dv
Pliuuid
s= '-'-'' Eq. 1-6 -dy dy
Pwater where the constant of proportionality k is called the dynamic of
absolute viscosity denoted as p.
In gases, the standard reference to calculate the specific gravity is the density dv
T = Ll-
'dy
of air.

-- Pg*
LO-t-/
Pair

For water at 4oC: where:


y = 52.41b/ ftt - 9.81 kN/mt r = shear stress in lb/ fP ot Pa
p = 1.94 slugs/ft3 = 1000 kg/m3 p = absolute viscosity in lb sec/ft2 (poises) or Pa-sec.
s=1.0 y = distance between the plates in ft or m
U = velocity in ft/s or m/s

VISCOSIW, p (MU)
The property of a fluid which determines the amount of its resistance KINEMATIC VISCOSITY v (NU)
shearing forces. A perfect fluid would have no viscosity. Kirrematic viscosity is the ratio of the dynamic viscosity of the fluid, p, to its
rrr.rss density, p. .
Consider two large, parallel U
Area = A
plates at a small distance y
apart the space between
them being filled with a fluid.
Consider the upper plate to
{
be subject to a force Fso as to where:
move with a constant velocity p = absolute viscosity in Pa-sec.
U. The fluid in contact with p = density in kg/m3
fixed plate
CHAPTER ONE CHAPTER ONE
6 Properties of Fluids
FLUID MECHANICS
Properties of Fluids 7
& HYDRAULICS

Table 1 - 2: Common Units of Viscosity Capillarity


Svstem Absolute, p Kint'nr.rtic, r,

lb-sec/ftr
English (slus/ft-sec) ftz/ sec
dyne-s/cmz cm2/ s
Metric
(poise) (stoke)
Pa-s
S.I m2/s
(N-s/mz)

Note:
L poise = 1. dyne's/cmk 0.L Pa-sec (1dyne = 10-s N)
1 stoke = Q.QflQl pz/g

cohesion
(a) Adhesion > cohesion (b) cohesion > adhesion

SURFACE TENSION o (SIGMA)


capiilaity (Capillary nction) is the name given to the behavior of the liquid in a
The membrane of "skin" that seems to form on the free surface of a fluid is thin-bore tube. The rise or fall or a fluid in a capillary tube is caused by
due to the intermolecular cohesive forces, and is known as surface tension. eurface tension and depends on the relative magnitudes of the cohesion of the
Surface tension is the reason that insects are able to sit on water and a needle is liquid and the adhesion of the liquid to the walls of the containing vessel'
able to float on it. surface tension also causes bubbles and droplets to take on
Liiuids rise in tubes they wet (adhesion > cohesion) and fall in tubes they do
a spherical shape, since any other shhpe would have more surface area per noi wet (cohesion > adhesionl. Capillary is important when using tubes
unit volume. emaller than about 3/8 inch (9.5 rnm) in diameter'

.rrt..: . t .' 4o cos 0


Pressure inside a Droplet of Liquid:
yd
. . 4O, :, r',,
q P,=.,
7. . , For complete wetting, as with water on clean glass, the angle 0 is 0o. Hence
the formula becomes
where:
o = surface tension in N/m n= 9 8q.1.-12
I d = cliameter of the. droplet in m ,'( d
p = g.lU(' pressure in I)a
where:
h; capillary rise or dePression in m
y = unit weight in N/m3
d = diameter of the tube in m
o = surface tension in Pa
8
CHAi'TER ONE
Properties of Fluids
|LUID MECHANICS
I HYDRAULICS
CHAPTER ONE
Properties of Fiuids I
Table 1 - 3: Contact Angles, Ar.
0
E"=iEEl
ub
sffarn
=4aY Eq.1-15
Materials Angle;0 ,
tlV
mercury-glass 140.
water-paraffin 107"
.''
water-silver -dpr
orEr= Eq'1-15
kerosene-glaqs
900
26 ffi
glycerin-glass 190
water-slass 0o
ethvl alcohol-slass 0o
PRESSURE DISTURBANCES
Pressure disturbances imPosed on a fluid move in waves. The
velocitY or
Eelerity of presswe wave (also known as acoustical or sonic ttelocity) is
CoMPRESSTBTLTW, p rxpressed as:
'as
Compressibilify (also known the coeffcient of compressibility) is the fractional
change in the volume of a fluid per unit change in pressure in a constant-
temperature process,

PROPERTY CHANGES IN IDEAL GAS


For any ideal gas experiencing any Process, the equation of state
is given by:

where:
AV= change in volume
V = original volume when temperature is held constan! Eq. 1 - 1.8 rgduces to (Boyle's Lmo)
Ap = change in pressure
dV/V = change in volume (usually in percent)

when temperature is held constant (isotfuermal condition), Eq. 1 - 1"8 reduces


to (Cturle'iLmu)

BUTKMODULUSOFELASTTCTTY, EB j

The bulk modulus of elasticity of the fluid expresses the compressibility of the
fluid. It is the ratio of the change in unit pressure to the corresponding
volume change per unit of volume.
II .T
t
CHAPTER ONE
Properties of Fluids
FLUID MEcHANIcs
& HYDRAULICS
FLUID MECHANICS
& HYDRAULICS
CHAPTER ONE
Properties of Fluids
ll
For Adiabatic or Isentropic Conditions (no heat exchanged) Table 1 - 4: TYPical VaPor Pressures
i
FIUid kPa,20oC
mercurv 0.000173
turpentine 0.0534
water 2.34
ethvl alcohol 5.86
ether 58.9
butane 218
Freon-12 584
propane 855
ammorua 888
where:
pr = initial absolute pressure of gas
pz= hnal absolute pressure'of gas
Vr = initial volume of gas
Vz= hnalvolume of gas
Tr = initial absolute temperatuie of gas in "K fK ='C'+ 273)
Tz = final absolute temperature of gas in oK
k = ratici of the specific heat at constant pressure to the specific heat at
Problem 1'1
of 0.952 cu. m.
constant volume. Also known as adiabatic exponent. A reservoir of glycerin has a mass of 1',200 kg and a volume
y,
Find ie (a) weight, W, (b) unit weight, (c) mass density' P' and (fl specific
gravity (s).

VAPOR PRESSURE Solution


Molecular activity in a liquid will allow some of the molecules to escape the (a) Weight, W= M I
liquid surface. Molecules of the vapor also condense back into the liquid. The = (1,200X9.81)
vaporization and condensation at constant temperature are equilibrium Weight W=11,772N or 1'L.72 kN
processes. The equilibrium pressure exerted by these free molecules is known
as thettnpor y)ressure or snturati1n pressure. (b) Unit weigh! |=+
Some liquids, such as propane/ butane, ammonia, and Freon, have significant
-fi..nz
vapor pressure at normal temperatures. Liquids near their boiling point or 0.952
rr
that vaporizes easily are said to aolatile liquids. other liquids such as mercury, Unitweighg Y =12'366kN/mr
|rave insignificant vapor pressures at the same temperature. Liquids with low
vapor Lrr('ssure are used in accurate barometers.
lvl
I he tendc,nr v toward vaporization is dependent on the temperature of the (c) Density, p=
i
lrtluid. Borirrrr'rri('urs when the liquid temperature is increased to the point
tlr,rt the vaPor FI.('\\trc, is equal to the local ambient (surrounding) pressure. Density, ,=#
I lrr.n. a liquid's boilrr r;:. ir'n.lp(rrafure depends on the local ambient pressure, as p=1.26o,5kd*'
Density,
well as the liquid's tcnt{t,rrr'r tr, ,, ,r}:orize.
l2 CHAPTER ONE
Properties of Fluids
FLUID MECHANICS
& HYDRAULICS
FLUTD MECHANICS
& HYDRAUTICS
CHAPTER ONE
Properties of Fluids t3
Pelv Solution
(d) -
Specific gravity, r =
;#; (a\ .W =*8 = 22(9.75)
1'26A ;5 W= 214.5 N
Specific gravity,, =
1.,000
(D) Since the mass of an object is absolute, its mass will stiil be22kg'
Specific gravity, s = 1.26

Problem 1- 2 Problem 1 - 5
specific gril/ity of certain oil is 0.82. Calcurate its (a) specific weight, in what is the weight of a 45-kg boulder if it is brought to a place where the
fh9- acceleration due to gravity is 395 m/s per minute?
lb/ ft} and kN/m3, and (b) mass density in slugs/ft3 u"a tg7*.

Sqtution Solution
(n) Specific weighL y = y*"t., x s
w= Mg
*/s , lmin
Specific weight, y = 62.4x 0.82 = 51.168 lbft3 o
o=3g5
min 60sec
Specific weight, y = 9.81 x 0.g2 = g.044 kN/ms
8= 6.583m/s2
(b) Density, p =p*,t",x s W= 45(6.583)
Density, p = 1,.94x 0.82 = 1.59 slugsff W= 296.25 N
Density, p = 1000 x 0.82 = Bil}kglma

Problem 1- 6
Problem 1- 3
If the specific volume of a certain gas is 0.7848 m}fkg,, what is its specific
A liter of water weighs about 9.7S N. Compute its mass in kilograms. weight?

Sotution Sotution
W 1
Mur, = . Vr=1
I p

Mur, =
9'75 11.
. 9.81 ' v,
J=-
0.7848
Mass = 0.994k9
p=1.2742ke/*

Specificweight,y=pxg
Problem 1- 4 =]'.2742x9.81.
If an object has a mass of 22 kgat seh lever, (a) what wifl be G weight at a Specific weight, y =lLS N/*'
point where tle acceleration due to gravity g = 9.TS m/ s2? (b) What will be its
mass at that point?
/
FLUID MECHANICS
CHAPTER ONE
I5
l4 CHAPTER ONE
Properties of Fluids
FLUID MECHANICS
& HYDRAULICS & HYDRAULICS
Properties of Fluids

Problem 1- 7 Solution

-8 r
v
What is the specific weight of air at 480 kPa absolute and 21oC? DensitY, P =

Solution _ 13.7
y=pxg 9'81'
= \.397 kg/m'
o = :P
,RT where D
-.--^--^ R= tt-. or.
287 J/kg:K
- ^o-r 0
DensitY, P =
480 x 103 iT
- (205 + 101'325) x 103
287(21. + 273)
L'r, :
1 ,on
R@2+273)
Note: Patm = 101'325 kPa
p = 5.689 kg
Gas constant, R = 718'87 l1r1g - "K
Y=5.689x9.81
y = 55.81N/m3
Problem 1- 10
and a temperature of 30oC in a
Air is kept at a Pressure of 200 kPa absolute
Problem 1- 8 li00-liter container. What is the mass of air?
Find the mass density of helium at a temperature of 4 oC and a pressure of 184
kPa gage, if ahnospheric pressure is10L.92 kPa. (R = 2079I/kg . 'K) Solution

Solution o=
'RT
P

Densitv.
J'I o = RT
P 200 x 103
287(30 + 273)
P=Psase+Patm kglm'
P =2.3
= 184 + 101..92
P = 285'92kPa
1u1utt=PxV
I

I
T=4+273=277"K =2'3x ffi
.. 285.92x 103 Mass = 1.15 kg
ljenslw,
r' o =
i
' 2,079(277',)
Density, p = 0. 4955 kglm3 I
Problemt-ft
AcylindricaltankS0cm-indiameterandg0cmhighisfilledwithaliquid. tank is 40
- 'lrc tank and the fiq"ia-*"igL; d 420 kg' The weight of the empty
'f
Problem 1- 9 kN/m3'
k1i. What is the unifweight of the liquid in
At 32"C and 205 kPa gage, the specific weight of a certain gas was 13.7 N/ms.
Determine the gas constant of this gas.
l6 CHAPTER ONE
Properties of Fluids FLUID MECHANICS FIUID MECHANTCS CHAPTER ONE
l7
& HYDRAUI.TCS I HYDRAULICS Properties of Fluids
Solution
Solutlon
p=_M
V ^dP
' --
LR=
420 _ 40 dv/v
''= =84okg/mr
,=p8
i1ffiy dp=pz-pt
h=o
dp=p,
= 840(9.81) = 8240.4 N/ms
' '8.24 kN/mr dV = Vz- Vt
dV = -0.6%V = -0 006V

Drobtem 1- f2 Eo=- Pz
- o.oo6v =z.z
4 lead cube has a totar mass of g0 kg. 14/hat is the rength of its
/v
lead = 11.3. side?.sp. gr. of pz= 0.0132GPa
pz=13.2}NIPa
Solution
Let L be the length of side of the
cube:
Prcblem 1- 15
M= pv Water in a hydraulic press, initially at 137 kPa absolute, is subjected to a
80 = (1000 x 11.3) Ls
pressure of 11,6,280 kPa absolute. Using Es = 2.5 GPa, determine the
L = 0.792m = 19.2 cm
pcrcentage decrease in the volume of water.

lotutlon
Problem 1- t3 do
A liquid cornpressed in a container has a volume
of L liter at a pressure of 1
-= --..l_
ER
dv/v
MPa.ald a volume of 0.995 liter at u pr"rSr*
oi;;;; The bulk modul-us of
elasticity (Ea) of the Iiquid is: 2.5 x 1oe - -$1'6;280 -T37)x1'03
dv /v
Solution dV
= -o.o4o5
r -
E_D=--
dP z-'1, v
=_-
L= *esrt decrease
Ea = 200 MPa v

l
Problem - t4 Prcblem 1- 16
oC and 150 kPaa is compressed to 2 m3, (a)
what is
what pressure is required to reduce the vorume lf 9 m3 of an ideal gas at24
of water by 0.6percent? IBurk thc, resulting pressure assuming isothermal conditions. (b) What would have
modulus of elasticity of water, EB = 2.2 Gpa.
Lrccn the pressnre and temperature if the process is isentropic. Use k = 1.3.

L
-T
I
T8
CHAPTER ONE
Properties of Ftuids
FLUID MECHANICS
& HYDRAULICS
FLUID MECHANICS
& HYDRAULICS
CHAPTER ONE
Properties of Fluids t9
I

I Solution Problem 1'18


I

,[,wo is filled
For isothermal condition: large plane surfaces are 25 mm apart and the space between them
I
(a)
Pr Vt = pzVz Assuming the velocity gradiq:rt to be
I with a tfirria of viscosity p = 0.958 Pa-s.
150(e) = pz (2) to pull a very thin plate of 0.37 m2 area at
a etraighi line, what forie is required
pz= 675 kPa abs mm from one of the surfaces?
, .or.,,furrt speed of 0.3 m/s if the plate is 8.4

(b) For isentropic process:


Solution
pt Vtk = pzVzk
15010;t.r = p2 (2)t.t
F=Fr+Fz
1'
pz = 1,060 kPa abs
'
Lt=
u/v
-
T, u-_F/A
-
(pr)(k-l)/* ' u/y
[-tr,.] -_ puA-
T2
24.2?s - [f1,060\(1'3-1)/1'3
1s0,j
v
0.958(0.3X0.3n
Tz= 466.4oK or 193.4oC rr - 0.0156
=5.4N

0.958(0.3X0.3n
l
F-'"-
12 - = 12.56 N
0.0084
Problem L - t7
If the viscosity of water atz} oC
is 0.00402 poise and its specific gravity is a.97g F = 6.4+ 12.66
determine its absolute viscosity in pa - s and its kinernatic vis"cosity in m2/s F = 19.06 N
and in stokes.

Solution Problem 1- 19
Absolute viscosity: A cylinder of mm radius rotates concentrically inside a fixed cylinder of
125
viscosity of
130'mm radius. Both cylinders are 300 mm long' Deterrnine the
u = 0.00402 poise x 0.88 N-m is
the liquid which fills the space betwEen the cylinders if a torque of
ffi required to maintain an-angular velocity of 2n radians/sec. Assume the
p = 0.fi)0402 Pa - s
vel,ocity gtadient to be a staight line'
Kinematic viscosity:
.._ rr_ 0.000402
p (1000x 0.978)
v = 4.1.1. *19.2 ltz/s

L stoke
v= 4.1.1. x10, m2/ s x
' 0.0001m2/s
v= 4.1'1, x LO3 stoke

-
[-
CHAPTER ONE
20 Properties of Fluids FLUID MECHANICS FIUID MECHANICS CHAPTER ONE
2l
& HYDRAULTCS & HYDRAULICS Properties of Fluids
So!!rtion
[E]:- = o]
' u/v
T

Y = 0.005 m
Wsin0-Fr=0
F, = Wsin 0
l,.L =r co
U=0.785 F=rA F,= 176.58 sin 15o
LI = 0.125(2r)
U = 0.785 m/s tl
fixed cylinder [F,=tA=paA]
y = 0.005 m v
Torque = F(0.125) 176.5gsin 15o = 0.0814# (0.3)
Torque = tA (0.125)
I Ll = 5.61.4m/ s
0.88 = t [2n(0.12s)(0.3)] (0.125) tt1=5'61'4m/s
I
t = 29.88 Pa

29.88 Problem 1- 21
r, = dEEZdJdS
llstimate the height to which water will rise in a capillary tube of diameter 3
p = 0.1.9 Pa-s
mm. Use o = 0.0728 N/m and Y = 9810 N/m3 for water.
0.13 m

I Solution
Problem 1.26 Note: 0 = 90o for water in clean tube
An 18-kg slab slides down a r.5o incrined prang on
a 3-mm-thick film of oir
with viscosity p = 0.0814 Pa-sec. If the contact area
is 0.3 m2,/ find
Capillary rise,h= 4
!r terminal
^'rru the
velocity of the slab. Neglect air resistance 4(o'ol?9.,
Capillary rise,h=
Solution e810(0.003)
W = 18(9.81) = 176.58 N
Capillary rise,h= 0.0099 m = 9.9 mm

s
y = 0.003 m
Problem L - 22
Estimate the capillary depression for mercury in a glass capillary tube 2 mm in
diameter. Use o = 0.514 N/m and 0 = 1"40o.
plane
Solution

Terrninal velocity is attained when the sum


of all Capillary rise, h = -0.0059 m (the negative sign indicates capitlary depression)
mction is zero.
forces in the direction of
Capillary depression, ft = 5.9 mm
_T
CHAPTER ONE
22 Properties of Fluids
FLUID MECHANICS
FLUID MECHANICS CHAPTER ONE
23
& HYDRAULICS & HYDRAULICS Properties of Fluids
Problem 1- 23
What is the value of the surface tension of a small
Problem l'26
drop of water 0.3 mm in A sonar transmitter operates at 2 impulses per second. If the device is held to
diameter which is in contact with air if the pressure
within the droplet is 561 tlrc surface of fresh water (EB = 2.04 x 10e Pa) and the echo is recdived midway
Pa?
bctween impulses, how deep is the water?
Solution
Solution
4o
r=7 'l'he velocity of the pressure wave (sound wave) is:
ZI6
r;-
l"B
5bl =
0.0003 " 1p
o = 0.042 N/m
,= ^@- =7,4z.m/s
v 1000
Problem 1- 24
Since the echo is received
An atomizer forms water droplets 45 pm in diameter.
Determine the excess midway between imPulses, then
pressure within these droplets using o
= 0.0712 N/m. the total time of travel of sound,
t = Yz(0.5\ = 7/t sec and the total
Solution
distance covered is 2/r, then;
'4o
,d 2h=ct
4(0.0712\ ?Jt = 1.,428(1/t)
P= -#x =6,329Pa h=778.5m
45 10-6

Problem 1- 25 Problem L - 27
Distilled water stands'in a glaSs tube of 9 mm At what pressure will 80 "C water boil?
diameter at a height of 24 mm.
What is the true static height? Use o = 0.0242 (Vapor pressure of water at 80"C = 47.AkPa).
N/m.
Solution Solution
, 4ocos0
'-
Water will boil if the dtmospheric pressure equals the vapor Pressure'
h-=
yd
Therefore water at 80 "C will boil at 47.LkPa.
where 0 = 0" for water in glass fube

l1=ffi=o.oo336m=3.36mm

True static height = 24 _ 3.36


True static height = 20.d4 mm
CHAPTER ONE
24 Properties of Fluids CHAPTER ONE
FLUID MECHANICS 25
& HYDRA
& HYDRAULICS Properties of Fluids
Problem 1- 33
Problem l - 2g (n) 12 m3 of nitrogen at 30oC and 125 kPa abs is permitted to expand
It
lsothermally,to 30 m3, what is the resulting pressure? (b) what would the
H:1H*ilT'il"J;',t^t or 1 3-kg mass on a pranet
where the accererarion pressure and temperature have beenif the process t". t,rrj?:ifjy:;};
rs: 30 N

Problem l - 29
A verticar cyrindrical tank with Problem L - 34
a diameter
a depth of 4 m is fired 1.1 kN and 250 mm on an edge slid.es down an
;l[nT:::r,,_1i,,:t.",,*
*ut", ut io.C' rrtil *;d;;:#;i"
?!!2 ^and
50oC, how
A square block weighing
incline on a film of oil 6.0 pm thick. Assuming a linear velocity profile in the
rf
l
tl.r7-,,"i^o;;liil',fl::1".'rTj,;"ight or water at iorc ,"a'so.C is e7e oil and neglecting air resistance, what is the termin4l velocity of the block?
t
The viscosity of oil is 7 mPa-s. Angle of inclination is 20o.
I Ans:4.7 m3 Ant;: 5.15 m/s

i Problem 1- 30
A rigid steer container is partiary
fired with
Problem 1- 35
the liquid is 1,.23200 r. a liquid at 15 atm. ,he vorume
at pJ"*; #;o ,or, ,h" ,;t;;;
of Benzene at 20'C has a viscosity of 0.000551 Pa-s. What shear stress is required
1'231'00 L' Find
'the average"urrt riquid is
";.;" over the to deform this fluid at a strain rate of 4900 sr?
given range of pressure -oarr,rr of erasticity of the riquid Ans'.r=3'19Pa
return to its initial value. ii.tr,"
t"*n".Ini" ur*, compression il- arowed to
What i, ,f,J"""fil.ient of compressibilitu?
Ans: Ee =.t.g72Gpa; p
= 0.534 Gpa{ Problem 1- 36
Probtem A shaft 70 mm in diameter is being pushed at a speed of 400 mm/s through a
1- 3l
bedring sleeve 70.2 mm in diameter and 250 mm l,ong The-clearance, assumed
*u,", vapor at 350 kpa abs and uniform, is filled with oil at 20oC with v = 0.005 m2/s and sP. 8r' = 0.9. Find
I :x'."#l"rthrffi, 20.C if its gas constant
the force exerted by the oil in the shaft.
Atts: 987 N
l
Ans:2.S9kg/mz

Probtem I - 32 Problem L-37 .

Two clean parallel glass plates, separated by a'distance d = L.5 mm/ are dipped
ix##:'.?i;;1ff:H:,::iff"* and a temperature or 30"C
in a 500-L in a bath of water. How far does the water rise due to capillary action, if o =
0.0730 N/m?
Ans:'1..15kg Ans:9.94 mm
--7
CHAPTER ONE CHAPTER T\VO -a
26 Properties of Fluids |LUID MECHANICS
Principles of Hydrostatics Z I
I
I

I
HVDRAULICS

I
Problem I - 38

Chapter 2
I
l Find the angle the surface tension film leaves the glass for a vertical
immersed in water if the diameter is 0.25 inch and the capillary rise
is
inch. Use o = 0.005 lb/ft.

Principles of HYdrostatics
Problem 1- 39
what force is required to lift a thin wire ring 6 cm in diameter from a wr
surface at20"C? (o of water at 20'c = 0.072g N/m). Negrect the weight
of XHI"::"::I"T:: ;:i::lllll
acting at
*",,"u bv a riquid or sas on a bodv or
right angles to the surface uniformly in all
nng. turface, with the force
Ans: O.O274 rllrections.
I

inch
tn the English system, pressure is usually measured in pounds per square
(pni); in iit"rnatio.tul usage, in kilograms Per square centimeters-(kg/cm2)' or
in ttmospheres; and in the international metric system (SI), in Newtons per
t(lunre meter (Pascal). The upit atmosphere (atm) is-defined as.a pressure of
t,ilggZe kg/cm2 (1,4.6961b/inz), which, in terms of the conventional
mercury
lrart,metei, corresponds to 750 rnm (29.92L in) of mercury' The
qnit kilopascal
(kl'n) is defined ui u pr"ttrt" of 0.0102 kg/ cm2 (0.145 lblsq in)'

FASCAL'S LAW
l,tnrnl's law, developed by French mathematician Blaise Pascal, states that
the
in all directions and in all parts ofthe container. In
rl.(!$sure on a fluid ir uq"ut
i,lp,uru 2-L, as liquid flows into the large container at the bottom, Pressure
tubes above the container' The liquid
;ru8lr", the liquid equally up into the of the
rlrcs to the same level in aU of tne tubes, regardless of the shape or angle
Ittbc,

\-
.T
CHAPTER T\vO
28 Principles of Hydrostatics
FLUID MECHANICS i
ILUID MECHANICS
Principles of
GHAPTERT\x/O 20
HydrostaticstL
I & HYDRAULICS 3 HYDRAULICS '
IISOLUTE AND GAGE PRESSURES

0lgo Pressure (Relative Pressure)


fiage pressures are pressures above or below the ahnosphere and can be
' Hcaeured by pressure gauges oi manometers. For small Pressure differences, a U-
httre nranometer is used. It consists of a U-shaped tube with one end connected to
Hs container and the other open to the ahnosphere. Filled with a liquid, such as
Weter', oil, or mercury, the difference in the liquid sur{ace levels in the two
manornc'ter legs indicates the pressure difference from local atrnospheric
eontlitions. For higher pressure differences, a Bourdon gauge, named after the
Frerrch inventor EugEne Bourdon, is used. This consists of a hollow metal tube
Figure 2 - 1: Illustration of pascal,s Law wllh un ova.l cross section, bent in the shape of a hook. One end of the tube is
,l
€lttrct{, the other open and connected to the measurement region.
The laws of fluid mechanics are observabre in many everyda-y situations.
For
example, the pressure exerted by water at the botiom oi a prnd will be
the Atmospheric Pressure & Vacuum
same as the pressure exerted by water at the bottom of a mu&
narrower pipe,
provided depth remains constant. If a longer pipe filled with water is tilted Attturspheric Pressureis the pressure at any one point on the earth's surface from the
so
that it reaches a maximum height of 15 m, its water will exert the same welglrt of the air above it. Aancw.im is a space that has all matter removed from it.
pressure as the other examples (left of Figure 2 - 2). Fluids can flow ll lr impossible to create a perfect vacuum in the laboratoryi no matter how
up as well
as down in devices such ai siphons (rigit of Figure 2 - 2). Hydrostatic Edvnnced a vacuum system is, some molecules are always Present in the vacuum
force
causes water in the siphon to flow up and over the edge until.the bucket is eren, Even remote regions of outer space have a small amount of gas. A vacuum
empty or the suction is broken. A siphon is particurart/ usefur for emptying fstr also be described as a region of space where the Pressure is less than the
containers that should not be tipped. horrrral atrnospheric pressnre of 760 mm (29.9 in) of mercury.

lJrrrlcr Normal conditions at sea level:


1t^y,, = 21,66\bf fP
= "14.7 psi
= 29.9 inches of mercury (hg)
= 750 mm Hg
= 101.325 kPa

pl =p!=p3
Abrolute Pressure i
Figure 2 - 2: Illustration of pascal,s Alrrolute pressure is the pressure above absolute zero (anurum).

aus = Pgage
+ Patnr Eq.Z - 2

Note:
. Absolute zero is attained if all air is removed, It is the lowest possible pressur€ attainable.
Absolute pressur€ crn never be negative.
'. The smallest gage pressure is equal to the negative of the ambient atmospheric pressure.
30 CHAPTER TUTO
Principles of Hydrostatics
FLUID MECHANICS FLUID MECHANICS
CHAPTER TUTO
Principles of HYdrostatics
3l
& HYDRAULICS I HYDRAULICS

VARIATIONS IN PRESSURE
T-' etrnrtder any two points (1" hxz),whose difference
in elevation is lr, to lie in the
60 gage area a a'.d a length of L.
enelt of an elementary prir* having a cross-sectional
Standard 58.675 gage
atmosphere = 101.325 abs it must be in equilibrium'
[ilnce this Prism is at rest, all forces icting upon
I
I
v

-40 gage Current atmosphere = 100 abs Free liquid surface

pressures
60 abs Absolute zero = -101.325 gage
or -100 gage

All pressure units in kpa

Figure 2 - 3: Relationship between absolute and gage pressures

Note: Unless otherwise specified in this book, the term pressure signifies gage pressure.

MERCURY BAROMETER
A mercury barometer is an accurate and relatively
simple way to measure changes in atmospheric
pressure. At sea level, the weight ofthe atmosphere
forces mercury 760 mm (29.9 in) up a calibrated
glass tube. Higher elevations yield lower readings Figure 2 - 4: Forces acting on elementary prism
because the atmosphere is less dense there, and the or with
to liquid surface subject to zero gage pressure
thinner air exerts less pressure on the mercury, Note: Free Liquid Surface refers
atmospheric Pressure only' '

With reference to Figure 2 - 4:


w=t v
W=y (aL)

ANEROID BAROMETER trr, = 0l


In an aneroid barometer, a
partially evacuated metal drum
Fz- Ft = Wsin 0
expands i:r contracts in response Pza-fia=Y(aL)sin0 I
to changes in air pressure. A p2-P't=yLsin0 butLsin0=h
series of levers and springs
translates the up and down
movement of the drum top into
the circular motion of the
pointers along the aneroid 'fherefore; tlrc difference in pressure befioeen any kob points in a homog.eneous fluid
barometer's face.
Mltal drun trt rest is equnt to' the product of the
unit weight of the fluid (y) to tlu aertical distance
(ptrtlal v.cuum)
(l) behoeen the Points.
32
AIso:
"friJ:f;H;,_starics FLUID MECHANICS T\VO 2 2
CHAPTER
I HYDRAUI-ICS Principles of Hydrostatics J J

I ('unsirlcr the tank shown to be filled rvitl"r liquids of different densities and
willr air at the top under a gage pressure of p.0,, the pressure at the bottom of
lltr'tirnl< is:

Ir.,-, :

PRESSURE HEAD
I'r't,ssure head is the height "17" oI a column of homogeneous liquid'of unit
wt,iliht y that will produce an intensity of pressure p.

WWil;,:tar;
the product rf
;;r r'riff r,;:;;r;:i:;rf;r:;;r;

[f;-1"; *::P,,,T
that h = o, o and o in Fisure
,r,""iq. i_ ;*:3;fl,,1 2 _ 4 rie on
To Convert Pressure head (height) of liquid A to liquid B

lxB = bAs
A or lts = lxAp
A or hr = n
1ro^l Eq.2-9
I;,r;;:; #t the p re
s su r e

zontal plane
So Ps YB

in a homogerrcous
Jluid
To convert pressure head (height) of any liquid to water, just multiply its
Pressure below height by its specific gravity
layerc of Different
liguids
llwater = huqq;4 x S1i.rr;d 2 -10
CHAPTER T\vO T\VO
34 Principles of Hydrostatics FIUID MECHANICS ILUID MECI{ANICS
CHAPTER
Principles of
,E
Hydrostatics 5 )
6( HYDRAULICS I HYDRAULICS
MANOMETER
Itrpt ln Solving Manometer Probtems:
A ruanometer is a (rbe, usually bent in a form of a tJ,containing
a liquid 1, Decide on the fluid in feet or meter, of which the heads are to be
the surface of which moves proportionaly
Iil::":.f.t*^t:-l:,y,
of pressure. It is used to measure pressure.
to chanl cxpressed, (water is most advisable).
2, Starting from an end point, number in order, the interface of different
fluids.
Types of Manometer 3, Itlentify points of equal pressure (taking into account that for a

homogeneous fluid at rest, the pressure along the same horizontal plane
open Type - has an atnnospheric surface in one Ieg
o and is capabre
'----- -- of are equal). Label these points with the same number'
pressures.
measuring gage
4, Irroceecl from level to level, adding (if going down) or subtracting (if
Differential Type - without an atmospheric surface and capabre of going up) pressure heads as the elevation decreases or increases,
measuring only differences of pressure. ierp"ctirrety *ith due regard for the specific gravity of the fluids.
Piezometer - The simprest form of open
manometer. It is a tube tapped into a
wall of a container or conduit for the purpose
of measuring prJssure. ,*re
fluid in the container or conduit ris"r i^ tt i,
tube to ror* u"rrl" lil;;":':

Limitations of piezometer:
. Large pressures in the lighter liquids
require long tubes
' Gas pressures can not be meazured because
gi ,,ot form a free Problem 2 - 1
surface. "ur,
If n clepth of liquid of 1 m causes a pressure of 7 kPa, what is the specific
grnvity of the liquid?

tolution
Pressure, p=ylt
7 = (9.81x s) (1)
s= 0.714 ) SPecific GravitY

(a) Open manometer Problem 2 - 2


(b) Differential manometer
what is the pressure 12.5 m below the ocean? Use sp. gr. = 1.03 for salt water.

Solution
p=vh
p = (e.81x1.03)(12.5)
P =126'3kPa

h
7

CHAPTER T'fr/O
36 Principles of Hydrostatics FLU'D CHAPTER TWO
&
FLUID MECHANICS
& HYDRAULICS Principles of Hydrostatics 37
Problem 2 - 3
Ifthe pressure 23 meter below Problem 2 - 5
a liquid is 33g.445 kPa, determine
weight I, mass density p and its lf the pressure in the air space above an oil = 0.75) surface in a closed tank
(s is
, ,p".ifi. gr;rrit r. 1 15 kPa absolute, what is the gage pressure 2 m below
the surface?
Solution
(o) Unit weight, y Solution

P=vh P=Prurtur"+Yh
938.44s = y Psurtace= 115 - 101'325 Note: Put = 101.325 kPa
eg) = 13'675 kPa gage
y =L4.715 kN/mr Psurface

p = 13.625 + (9.81x0.7s)(2)
,r) Mass density, p = 28'39kPa
P
^- -
p- v
I
Problem 2. 6
^ _ 14.715x103
___
9.81 Find the absolute pressure in kPa at a depth of 10 m below the free surface oi

= 1,500 kglms oil of sp. gr. 0.75 if the barometric reading is 752 mmHg.

(c) Specific gravity, s


Solution
Pfl.rid Pnbs= Pntnt
+
,= Pgage

Pwater Patm=ftrhtr
= (9.81 x13.5)(0.752)
s= r1.500
1,000 P"t* = 100'329 kPa
s=L.5 pna, = 100.329+ (9.S1 x 0.75)(10)
Pnas=173.9kPa

Problem 2 - 4
If the pressure at a point in Problem 2 - 7
the ocean is 60 kpa, what is
below this point? the pressure 27 meters A pressure gage 6 m above the bottom of the tank containing a liquid reads 90
kpa. Another gage height 4 m reads 103 kPa. Determine thJspecific weight of
Solution the liquid.
The difference in Dresr
between anv two points in
riq"ia i, p, -';;:;!;'"t"te a Solution

pz=p+yh
= 60 + (9.81x 1..03)en P2-tu=Yh
pz= 332.82kpa tog-90=y(2)
y = 6.5 kN/ms
CHAPTER T\VO
38 FLUID MECHANICS FLUID MECHANICS CHAPTER T\vO 2GD
Principles of Hydrostatics
& HYDRAULICS & HYDRAULICS Principtes of Hydrostatics il
Problem 2 - 8
lolution
An open tank contains 5.g m of water covered
with 3.2 m of kerosene (y = g Since the density of the mud varies with depth, the pressure
kN/ma;. Find the pressure at the intqface
and at the bottom of the tank. should be solved bY integration
Sotution
dp=ydh
(a) ' Pressure at the interface
dp=(1.0+0.5h)dh
Pn=fxhr 'a
= (8)(3.2)
= o*o.snan
pe= 25.6kPa
Kerosene -----r
)+
00
)t
(b) Pressure at the bottbm r = 8 kN/m3 3.2 m 15
Pa =Z yh
o= 1,0h+O.25hz
= Yn,lq, + yp h* Water
' _l
I

o
y = 9.81 kN/m3
= e.81(s.8) + 8(3.2) = t10(5) + 0'25(5)'z1 - 0
pe = 82.498kpa P = 56.25kPa

Problem 2 - 9 Problem 2- LL
If atmospheric pressure is 9s.z.kpa and the
gage attached to the tank reads
ln the figure shown, if the atmospheric
mmHg,vacuum, find the absolute pressure 1gg pressure is 101.03 kPa and the absolute
iritt ir, the tank.
pressure at the bottom of the tank is
Solution 231 .3 kPa, what is the specific gravity
ptus=pmn+pgnge of olive oil?
Pg,rg, = Tm.rcury lxmercury
= (e.81 x 13.6)(0.188)
= 25.09 kpa vacuum
Ps s= -25.08 kPa
pax=95.7 + (-25.08)
paw= 70;62 kPa abs

Problem 2 - l0 Solution
The weight density of a mud is given
by y = 10 + g.Slt,where y is in
*'^kN/me and Gage pressure at the bottom of the tank, P = 231"3 - 101'03
ft is in meters. Determine the pr"essur",'i"
tpu, d;p;-;f;,;. Ga[e pressure at the bottom of the tank, p = 130.27 kPa
"t "
lp = zvhl
.P ='(u,hu,
* Yoho * Yu,hru t Yothoit
130.27 = (9.81x 13'6)(0.4) + (9.81x s)(2.9) + e.81(2'5) + (9'81 x 0.8e)(1.5)
s = L.38
40 Ff*?;l::H,!o,o,,,,,., FTUID MECHANICS
& HYDRAULTCS
FLUID MECHANICS
& HYDRAULICS
CHAPTER
Principles of
T\VO
Hydrostatics 4lI !
Problem 2 - 12 Probfem 2 - t4
If air had a constant specific weight of. 12,2 N/m3 and were incompressible, (irnrpute the barometric pressure in kPa at an alritude of 1,200 m if the
what would be the height of the atrnosphere if the atmospheric pressure (sea conditions a 21"C. Use
level) is 102 kPa? l,Icfisure at sea level is 101.3 kPa. Assume isothermal
ll - 287 Joule /kg-'K.
Solution Solution
Height of atmosphe re, h = L I:or gases:
v
102 x 103
12.2
Height of atmosphere,ll= = 8,360.66 m o=
"RT
P

_0
287(2"1 + 273)
- 13 (CE Board May 1994)
2
p = 0.0000L185 p
Assuming specific weight of air to be constarit at 1.2 N/m3, what is the
approximate height of Mount Banahaw if a mercury barometer at the base of dp = -(0.00001185 p)(9.8i) dft
the mountain reads 654 mm and at the same instani, another barometer at the
top of the mountain reads 480 mm. lL : o.ooorru, nn
p

Solution p 1200

[4t -o.ooorru, ln,,


=
Jp
1gl.lx103
J
0

1p 11200
lnp l_.-^ .^, = -0.0001163,, .lu
J 101.3xlo'

Inp - In (101.3 x 103) - - 0.0001i63(1200 - 0)


ln p = 11.385

P = e17.386

P = 88,080 Pa

Pact-ptop=fh
(Y, h,,)uo,to. - (yn, h*)op = (y h)ai,
(9,810 x 13.6X0.554) - (9,810 x18.6)(0.48)=t/tu
h=7,934.53m
lT-

CHAPTER T\VO
42 Principles of Hydrostatics ILUID MECHANICS +3
I
& HYDRAUTICS
l,r Problem 2 - 15
I
Convert 760 mm of mercury to (a) oil
IfOblam 2 - 18 (CE November 1998)
of sp. gr. 0.g2 and (b) water. piston B is 950 sq. cm'
Fllton A has a cross-section of.1,200 sq. cm while that of
!
l

are
wlth the latter higher than piston Aby L15 m. If the intervening Passages
I

Sotution
fllled with oil whose specific gravity is 0.8, what is the difference in Pressure
(a) 1611 = lrmercury
ncrc ry between A and B.
I 5 oit
tolutlon
= 0.76#
l'^'PB = lolh
I
hoit='1.2.505m of oil = (9,810 x 0.8)(1.75)
lr,t - Pe = 13,734 Pa
(b) fr*r,", = hnre.cury smercury
,

lr = 0.76(13.6)
l
h*ater ='1,0.34m of water

lr
l
Probtem2-16@ 1200 cm2

i A barometer reads 750^mmfS urra a pressure


gage attached to a tank reads
850 cm of oil (sp'
$. 0's0). w^"at is *," lurri"t" pressure in the tank in kpa?
I

Solution ln the figure shown,


paas=patn+pgage dctermine the weight W
= q_q1 xB.6)(0.76) + (e.81x 0.8)(8.s) that can be carried bY the
P"u = 168.1 kPa abs 1.5 kN force acting on the
piston.

Problem 2 - tz
A hydraulic press 'is used to raise an gO-kN
cargo truck. If oil of sp. gr. 0.g2
acts on the piston under a pressure
of 10 Mpi wtat aiamlie.-of piston is
required?

Solution Solution
Since the pressure under the piston Since points 1 and 2lie on the 300 mm Z
is uniform:
Force=pressurexArea same elevati on, Pl = P2
80,000 = (10 x 103)
D=0.1.m=1.(X)mm
tD2 1.5_w
t(o.m)2 t(0.3)2
W= 150 kN
CHAPTER T\vO
44 Principles of Hydrostatics FIUID MECHANICS MECHANICS CHAPTERT\VO A"\
6. HYDRAUTTCS 'tUID
& HYDRAULICS Principles of Hydrostatics ' r
Problem 2 - 20
A.drum 700 mm in diameter and lolutlon
indiarneter, il;il filled with
to the top.
poured into the pipe to exert a
H;;;,:fKHiiflI.#JJi?11,7 W=44kN
=44kN
foice of 6500 N on the top of the = 0.323 m2
drum?
Solution
Force on the top:
F=pxArea h=4.6m a = 0.00323 m2

6500=px
tV002_202)
p = 0.015904 Mpa
1t= 1.6,904pa

lp=yhl Oil, s = 0.78 Oil, s = 0.78


1.6,904 = 98701r
h='1,.723m lpz-pt=y"hl
D1 =
rF
Weight=yxVolume n
- 0.00323
=9810 x f,p.Az)z(1.725) p=309.6 F .(kPa)
Area on top
trVeight = 5.31 N w44
t'z A o.zzg
700 mm @
pz= 136.22kPa
Problem 2 4t
The figure shown shows a setup 136.22-309.6 F= (9.81x 0.78)(4.6)
with a vessel containing a plunger
cylinder. What force F is requirei and a F = 0.326 kN = 326 N
," [rfr*" ar," weight of the cylinder if
weight of the plrrnger is negligible? the

Cylinder Problem 2 - 22
W=44kN
A = 0,323 m2
l'he hydraulic press shown is filled with oil with sp. gr.0.82. Neglecting the
weight of the two pistons, what force F on the handle is required to suPPort
the 10 kN weight?

4.6 m
46 CHAPTER T\YO
FLUID MECHANICS FTUID MECHANICS
CHAPTER T\vO a7
I
Principles of Hydrostatics
6. HYDRAULICS & HYDRAULICS Principles of HYdrostatics ' '
Solution of gasoline'
lilnce the gage reads 'FIJLL" then the reading is equivalent to 30 cm
Since points 1 and 2lie on the same
elevation, then; Reacling (pressure head) when the tank contain
P't = Pz
F'=F' 7s water = g + Zfu)cm of gasoline

. A"t A2 Then;
L0 F,
Y+2# =30
U = 27'06 cm
+(o.o7qz te.o2q2
Fz = 1.11kN
I
@November20oo)
lxMo=ol Ibr the tank shown in the Figure, ht = 3m and lrg = 4 m. Determine the value
F(0.425) = Fz(0.025)
L

of ln.
r(0.425) = 1.11(0.025)
I
F = 0.0654 kN
i F=65.4N

FBD of the lever arm

Problem 2 - 29
The fuel gage for a gasorine.(s!- gr. 0.6g) Solution
= tank in a car reads proportionar to
its bottom gage. If the tank is so cm deep an
accidentaly 2 Summing-up Pressure head
cm of water, how many centimeters of gasoline "or,t#ir,'uted,with
does the tar.,t act rutty contain from L to 3 in meters of water
when the gage erroneously reads "FUL?,?
L +t4(0.84)-*= b
Sotution yY
0+0.84h2-$-3\=0
hz=1.19rlr

r
=l 30 cm
l_
"Full"
r
ll 48 CHAPTER TWO
FLUID MECI{ANICS CHAPTER T\vO
Hydrostatics 40
Principles of Hydrostatics FIUID MECHANICS t'
1
& HYDRAULICS & HYDRAULICS Principles of

lr
li
Problem 2 - ZS (CE Board May 1992)
In the figure shown, what is the static pressure
Froblem 2 - 26
I
in kpa in the air chamber?
For the manometer shown, -T
I
determine the pressure at the
I
center of the pipe. 1m
I

I'
l

'{r Mercury, s = 13.55


.5m
I
il

th

ril
l_
li '
rI
I

irr ,

ltr
Solution
ri
lr'
The pressure in the air space
Sotution
;t equals the pressure o., th" surface
il of oil, p3. Sum-up pressure head from
i

1 to 3 in meters of water:
i
i

Pt=o & + 1(13.55) + 1.5(0.8) = Pt.


v v
Pz= fn, hn,
= 9.81(2) o+tE.75= lL
v
pz=19.62kPa
s = 13.55
t:
Z! =1g.7sm,of water
"{
Pz-Ps=Yoh,
I
7e.62-ps= (e.81. x 0.80)(a) ps= 74.75(9.81\
I
pt=:17.77 kPa ps=744.7 kPa

I Another solution:
I
i,
Sum-up pressure head from 1 to 3 in meters of water:

?Yy* r- 4(o.so; = Ps

0 +2-3.2= Pt
9.81
pt= _t't.77 kpa

,i-..
'-
r
so CHAPTER T\x/O T\vO
l Principles of Hydrostatics FLUID MECHANICS
& HYDRAUL'CS
;LUIP MECHANICS
I HYDRAULICS
CHAPTER
Principles of HYdrostatics rtr II
Problem 2 - 27 (CE Board November 2OOf)
lroblom 2 - 28 (CE MaY 1993)
Determine the value of y in the manometer
shown in the Figure. Itr llrr: figure shown, when the
futrncl is empty the water surface
-T'-
b nl point A and the mercurY of
1m
rp. gr, 13.55 shows a deflection of
lc- 15 cu1. Determine the new
I rlt,flcction of mercury when the
3m
Irrrrnel is filled with water to B.
I

+_
1m
J

Solution

30cmo
Solution
-T-
Sumrning-up pressure head from
1m
A to B in meters of water:
+
?*
yf re.t)+ 1.s - y('13.q = PB I

I
3m

g# . 3.e _tz.ey = Pnv


1m
where pa = 0 J
T-
0.15
y = 0.324m

Mercury
s=13.55
Figure (a): Level at A Figure (b): Level at B

Solve for y in Figure (a):


Sum-up pressure head from A to 2 in meters of water:
oa +!t-0.15(13's5)= la
v-Y
0+y-2.03=0
y=2'03m
CHAPTER T\vO
52 Principles of Hydrostatics IIUID MECHANICS CHAPTER T\vO E 2
Hydrostatiqs J J
j

& HYDRAULICS S I.IVDRAULICS Principles of


In Figure (b):
When the funnel is filled with Sum-up pressure head flom 2 to m in meters of water:
water to B, point 1 wiJl move down
with the same value as point 2 to t, Pz +yqt3.6)-y- P"
moving
";;';, vy
Sum-up pressure head from B
to 2,:
P'*o.g*vtx- '(r+0,15+r403siq=P2'
13.5y - x= ffi Eq.(1)

T , ln Figure (b):

0+0.80 + 2.03.+ x -27,.tx


7 Sum-up pressure head from 7 to m' in meters of water:
-2.03 =0
26.1 x = 0.80 Pz' * (o.zsin0 + y + 0.2)(13.6) - + o.z) = ?nl
1x
r=0.031.m=3.l.cm YY
O + 2.72sin 0 + 13,6y + 2.72 - x- 0.2= +#
New reading, R = 15 + Zx + 2(2.1)
= 1,5
Eq (2)
I New reading, R = 21.2 cm 13.6y -r = 8.183 - 2.72 sin 0
I

113.6y-x=13.6y-xl
Problem 2 - 29 8.183 - 212 sine = ,*
Th^e_ presr,rre at point sin 0 = 0.3852
n in the figure
shown was increased from 70 0 = 22.66"
kp"a to
105 kPa. This causes the
top fur.i oi
mercury to move 20 mm in the
sloping
tube. What is the inclination, 0? Problem 2 - 30
A closed cylindrical tank contains 2 m of water, 3 m of oil (s = 0.82) and the air
nbove oil has a pressure of 30 kPa. If an open mercury manometer at the
bottom of the tank has 1 m of water, determine the deflection of mercury.
Solution
Solution
Sum-up pressure head from
1 to 4 in meters of water:
?nir *3(0.82) + 2+ 1. - y(13.6) =
P+

v v

ffi +2.a6+3-i3.6y=0
ll= 0'526m

Figure (a)
Figure (b)

In Figure (a):
CHAPTER Tlil/O
54 Principles of Hydrostatics
FLUID MECHANICS FLUID MECHANTCS
& HYDRAULICS
Problem 2 - 31
ln lilgure (b):
The U-tube shown is 10 mm in diameter Summing-up pressure head from 1 to 3 in mm of water:
and contains mercury. If 12 ml of water is Pl + 1s2.8-
poured into the right-hand leg, what are R(13.6) ='{a
yv
the ultimate heights in the two Lgs?
R = 11.24 mm

ln Eq. (2):
11..24+7s.=240
r = 114.38 mm

Ultimate heights in each leg:


Right-hand leg,hn=h+ x
= 152.8 + 114.38
Solution
Right-hand leg, hn = 267.78 mm
'Solving lor h,
figure b):
(see

Volume of water = (+$)ra = 12 cm3 Left_hand teg, trt =


* Note:1ml =1cm3
lr.rl .114.38
h =15.28cm = 152.8 mm
Left-hand leg,hr= 125.62 mm

quantity of mercury before and after water is poured


?1"*.*
remain the same, then;
Problem 2 - 32
120(3)=R+x+120+x Gage
For a gage reading of -17.LkPa,
R+ys=249 ) Eq. (1) determine the (a) elevations of
the liquids in the open-
piezometer columns E F, and
G and (D) the deflection of the
mercury in the U-tube
manometer neglecting the
weight of air.

El.8m

Figure (a) Figure (b)


56 CHAPTER TWO FLUID MECHANICS CHAPTER TWO EA
I Principles of Hydrostatics & HYDRAULICS
FLUID MECHANICS
& HYDRAULICS Principles of Hydrostatics il
Solution
Gage Column G
Sum-up pressure head from 1 to g in meters of water;

L- - t,r(t.o) = ?
yY s(0.4+
+ a(1)

1- # *2.1.+4-1.6h2=0
3m ln=2.7r^
Surface elevation = 8 + Its

i
4m
Surface elevation = I + 2.72 = 70.72 m

Deflection of mercury
Sum-up Pressure head from 1 to 5 in meters of wateri
Pr + Zp.4 +4+4 - ha(13.5\ = lL
1 YY
4m
I
# *10.7-13.6hE
pr = pair = '17.L kPa lk = 0.514 m
t ---v
El.4 m

Problem 2 - 33
An open manometer attached to a pipe shows a deflection 9f-fSO lmng with
Column E
the lower level of mercury 450 mm below the centerline of the pipe carrying
Sum-up pressure head from L to e in metes of water; water. Calculate the pressure at the centerline of the pipe'
P, *rrr(o.n=P,
y'y Solution
t# * h1(o'7)=s
h=2.5m
Surface elevation = 15 - hr
Surface elevation = 15 - 2.5 = 1.2.5 m

Column F
Sum-up pressure head from 1 to/in meters of wateri Sum-up pressure head from 1
3 in meters of water;
Pl- +g(0.7)-n0=L
yrl L * o.Es- 0.15(13.6) = i
T
-01# * 2'1'-h2=s
lu= g.r\,
Pt *0.45-2.04=o
^ = 12 + lg
Surface elevation
9.81
pr = 15.6 kPa
Surface elevation = L2 + O.\SZ = 1Z,3ST m
r" CHAPTER T\X/O T\yO iltro
58 Principles of Hydrostatics FLUID MECHANICS rLUID MECHANICS
C|-IAPTER
Principtes of HYdrostatics
& HYDRAUTICS & HYDRAULICS
Problem Z - g4
Solution
For the configuration shown, calculate the
m
weight of the piston if the pressure gage @) Gage liquid = mercury, h = 0.1
reading is 70 kpa. Sum-up Pressure head from
1 to 4 in meters of water;

Pt + x + h - h(13.6) - x - 1.5 = !-:-


-Pq
Y

Pt -Pq = 1.5 - 0.1 + 0.1(13.6)


v v
Pt -P+ =2.76mof water
Sotution v v

Sum-up pressure head from


(b) Gage liquid = carbon tetrachloride
A to B in meters of water;
reading, It = ?
Po -t1o'se1= P'
Yy Sum-up pressure head from 1 to 4 in meters of water;

086=# lL 1 v + tt - \1..59) - x - L.5 = b


k
pa = 78.44kPa
yY
Pr -Pq =1.5+0.59/r
vv
Weight = Fo Fa=paxArea Pt -?+ =2.76m)from(a)
where
= Pn x Area vv
=78.44x f,(1), 216=1..5 + 0.59h
Weight = 61.61 kN h=2.736m

Problem 2 - 35 Problem 2'36


Two vessels are connected to a differentiar. In the figure shown, determine Air, p = 175 kPa abs
manometer using mercury, the the height lr of water and the -Eft':==
connecting tubing being filled with water.
lower irr elevation than-the other. (a) If the
The higher pr"rrrr""r*ser is 1.5 m
mercu"ry .*ai"fi, i;r; ffi;
gage reading at A when the
absolute pressure at B is 290
T--
h
o
oo .Water
is the pressure head difference in meters I

of water? (b) If carbon tetrachrbride kPa.


G = 1'59) were, used instead of mercury, what wourd be
reading for the same pressure difference
?
v! the
rrls Imanometer +--
700 mm
o
o

l1q
w
i

CHAPTER T\YO
60 CHAPTER TWO ,- ,.
Hydrostatics (, I
Principles of Hydrostatics TIUID MECHANICS
3 TIYDRAULICS Principles of
Solution
Sum-up absolute pressure head Sum-up pressure (gage) head from 1 to 4 in meters of water;
from B to 2 in meters of water; Pt +xp.9\+1.3(0.e) -1.3(13.6)= b
!2 -o1og.e\-n= Pz
tla T-----:- Yv
yy I 40 *0.9x-15.51=0
h
9,81
ffi -o.sz-n= L75 I
r = 13.81m
+---
9.8-1
h =2.203 m
700 mm
J,.' Then,r+y=28.42m
i

I
I Sum-up absolute pressure head
ftom B to A in meters of water; 2-38 Oil, s = 0.85

I lL -o.z(ts.o) * 0.7 = l?or the manometer setup shown,


tl
? determine the difference in Pressure
bctween A and B.
I ffi -o.sz* 0.7 = h
d
J
pa = 203.5 kpa abs

,t

ll
Problem Z-E
i1
In the figure shown, the atrnospheric
pressure is 101 kpa, the Solution
gage
reading atA is 40 kpa, and the
vlpir a+0.58=y+1.7
pressure of alcohol is .12
tpu x-y= 1.02m ) Eq. (1)
absolute. Computex+y.
Sum-up pressure head from A to B
in meters of water;
Po -*-0.68(0.85) *y= b
yv
Pa Ps =x-rt+0.578
--. )Eq.(2)
Solution vv
fumlup absolute pressure head from Substitute x - y = 1..02 in Eq. (1) to Eq' (2):
1 to 2 in meters of water;
Pa Pe -n,.02+ 0.578
-
l 'v0sl = b vy
Pa-Pa -",
40 + 101-o'sv= 9.81
- ,.598
72

y
ls1 # PA- Pr = 15.68 kPa
= 74.67 m
CHAPTER T\VO CHAPTER T\YO
62 Principles of Hydrostatics
FLUID MECHANICS FLUID MECHANICS
Principles of Hydrostatics
63
& HYDRAULICS & HYDRAULICS

Problem 2 - 39 Problem 2 - 40
A differential rnanometer is In the figure shown, the
attached to a pipe as shown. deflection of mercurY is initiallY
Calculate the pressure 250 mm. If the Pressure at A is
difference between points A increased by 40 kPa, while
and B. maintaining the Pressure at B
constant, what will be the new
mercury deflection?

Solution

Solution

Sum-up pressure head from A to B in meters of water;

-vQsl- 0.1(13.6) + 0.1(0.e) + y(o.e) = lL


ff
Pt - Ps =0.1(13.6)-0.1(0.9)
yy
Pa-Pa =L.27rn
9.81.
PA-pB=12.46kPa
Figure (a) Figure (b)

In Figure a, sum-uP Pressure head from A to B in meters of water;


lL - o.e- 0.2s(13.6) + o.2s + 2.1= ry
v
Pe -PB =1.65mofwater
v"l
V:
CHAPTER T\X/O
64 FLUID MECHANTCS iLUID MECHANICS CHAPTER TWO , ?
Principles of Hydrostatics
& HYDRAULICS 3 HYDRAULICS Principtes of Hydrostatics O)
In Figure b, pA' = pt + 40
Sum-up pressure head from A, to B in meters of
lolutlon Air
water; Kerosene, s = 0.82 s = 0,0012

?-'v p.u- x) - (0.2s + 2x)L3.6 + (2.35 * *) = ,


3

rDr +40
^'-
Benzene l-
-'-- *= lLy
- 0.6 + x -2.4-27.2x + z.Zs + s = 0.88 E
v E
Im tr
E
o
. lL o
+-
? #'1"6s-2s'2x=
150 mm
Pa -Pa
yy =2s.2x-2.4?s B* PA
.'l
- P,
y
=1..6s
tl

'1,.65=25.2x-2.4?S
x = 0.162m = 162 mm
Sum-up pressure head from A to B in meters of water;
New mercury deflection = 250 + 2x = 250 + 2(162) b + o.z(0.s8) - 0.0e(13.5) - 0.31(0.82) + 0.25 - 0.1(0.0012) = Pp
,|
New mercury deflection = 574 mm v
Pa - Ps = 1.0523 m of water
Problem 2 - 4l
vv
Pa - Pa = 9.81(1.0523) = 10.32 kPa
In the figure showry determine the difference in pressure
between points A
and B.
Problem 2 - 42 (CE Board)
Assuming normal barometric pressure, how deep in the ocean is the point
where an air bubble/ upon reaching the surface, has six times its volurne than
it had at the bottom?

$olution
Applying Boyle's Law
(assuming iso thermal condition)

lpVr= pzVz)
p=107.3+9.81(1,03)r,
fi=101.3+1.0,7041r
Vt= V
pz=101.3+0=101.3
Vz= 6V
(101.3 + 70.704h)V = 101.3 (6 V)
10.104 ft = 101.3(5) - 101.3
h = 50.13 m
_V
t't

CHAPTER TUTO
66 Principles of Hydrostatics FLUID ILUID MECHANICS
&
Problem 2 - 4l
A vertical tube, 3 m long, with one end Slnce the pressure in air inside the tube is uniform,
closed is inserted vertically, with tlren p,, = p6= 20.0124kPa
:::ljll
connected i:yl
to the li:
upper
y,,",.. such a d;p,h ;i.;;;;;; il,"","
t1,.1. o1
: end of the Pc=T*h
tube reads 150 mm of *ur.-ri.y. Neglectr 20.0124 = 9.81h; h = 2.04 m
alsuming normat cond.itions, how far is
I#::lT:i:l1.yd
the tube below the water surface in
the tank?
the,lower end
Then; x=lr+ y=2.04+0.495
x = 2.535 m
Solution

I
Problem 2 - 44
0.1s m
{r A bottle consisting of a cylinder L5 cm in diameter and 25 cm high, has a neck
rl, which is 5 cm diimeter and 25 cm long. The bottle is inserted vertically in
water, with the open end down, such that the neck is completely filled with
p

3m,
water. Find the depth to which the open end is submerged' Assume normal'
I

t'
ll barometric pressure and neglect vaPor Pressure'

d Solution 15cmO

Applying Boyle's Law


Applying Boyle,s Law: pVt = pzVz
lir prVt= pzVz
Before the bottle was inserted:
Before the fube was inserted; Voluine of air:
Ab-solutepressure of air inside, pr
= 101.3 Vr= I (15)'? (25) + f (s)'z (2s)
Vt= 4,908'74cm3
When the tube was inserted; Absolute pressure in air:
Absolute pressure o{ air inside, pz 10.t p = 701"325
= .3+ 9.g1(13.6X0.15)
Absolute pressure of air inside, 'pz 121.i1,
= kpa
Volume of air inside the tube, Vz= (3 _ When the bottle is inserted:
y)A
Volume of air:
[prV= pzVz] Vz= t G5), (25)
101.3 (3 A) = 12]..st
IQ _
0 e) Vt= 4,417.9 cm1
3-Y=2.50s Pressure in air:
y = 0.495 m pz= 7A1..325 + 9.87 h
From the manometer shown;
lpVt= pzVzl
Pb=f,rhu, 101.325(4 ,90874) = (101'325 + 9.81. h)(4,417.9)
= (9.81 x 13.6)(0.15) 101.325 + 9.8'1. h = 11"2.58
pt = 20.0124kpa h = 1..15 cm
x=ht25=25.15cm
T
CHAPTER T1T/O
68 Principles of Hydrostatics FLUID MECHANICS
FIUID MECHANICS CHAPTER T\vO AO
& HYDRAULICS
I HYDRAULICS Principles of Hydrostatics v '
Problem 2- 45
t-
A bicycle tire is inflated at sea level, where
kPaa and the temperature is
the atmospheric pressure is 101.31
2'r. "C,' to 445kpa. Assuming
lsupplementary ProUem
the tire do", ,otl
is the gage pressure within the tire
lllu"d:,what
where the altitude is epO, m, atmosphuri.-prurrr.e on the top of a mountain, Problem 2 - 46
temperature is 5 oC. is 47.Z2kpaa, and the A wcrrtlrer report indicates the barometric pressure is 28.54 inches of mercury.
Wlrnt is the atmospheric pressure in pounds per square inch?
Solution Ans:14.02 psi

PtVr='--
P,>V,
Tt T2 Probfem 2 - 47
l'lrt' tube shown is filled with oil. Determine the pressure heads at B and C in
At sea level,
rrrt'[t'rs of water.
Absolute pressure of air, p1= 101.3 +
445 Pa
Absolute pressure, pr = = 546.3kpaa Ans'. = -2.38 m
Volume of ah, V1= l/
Absolute temperature of air. T1 2.2m Pc

-t-
= 21, + 27g = Z94oK = -0.51 m
I
v
On the top of the mountain:
Absolute pressure of air, p2 = 47.22 +
a 0.6 m
Since the tire did not expand, uol,rmJ
of air, V2 = l/ J
Absolute temperature o7 air, Tz= S +
Z7Z = 27g,K
Oil, s = 0.85
, PtVt P>V,
r -I
'T1Trl =-1-

s = 0.85
s46.s(vi _ g7.zz+p)v
294 --2?T Problem 2 - 48
47.22+p=51.6.57
p - 469.35 kpa For the tank shown in the figure, comPute the pressure at points B, C, D , and E
in kPa. Neglect the unit weight of air.
Ans pe = 4.9; pc = po = 4.9; Pe = 21.54
--"

70 CHAPTER T\x/O
FTUID MECHANICS rLUID MECHANICS
CHAPTER TWO a 'tI
Principles of Hydrostatics Hydrostatics '
& HYDRAUTICS & HYDRAULICS Principles of

Problem 2-49 ?

Problem 2 - 52
A glass U-tube open to-the atmosphere at both is a
contains oil and water, determinethe specific
ends is shown. If the u_tube A r.ylinc{rical tank contains water at a height of 55 mm, as shown. Inside
gravity of ,,,rnll np".t cylindrical tank containing cleaning fluid (s'g' = 0'8) at-a.height
the oil. Ir'
Assume the cleaning
Ans:0.96 l,lr,, pr"rsure ps = 13.4kPa gage and pc = 13.42 kPa gage.
l'lrrlti is prevented fro- -l.,Ittg to the top of the tank' Use unit weight of
wrrtcr = b.Zl tN7*r. (a) Determine the pressure pA in kPa, (b) the
value of lt in
rnrn, itnd (c) the value of y in millimeters'
Ans: (a) 12.88; (b\ 10.2; (c)'101

Problem 2 - 50
A glass 12 cm tall filled with water is inverted.
The bottom is open. what is
the pressure at the closed end? Barometric
pressure is 101.325 kpa.
Ans: 100.15 kpaa

Problem 2 - 51
In Figure 13, in which fluicl will a pressure of 700 kpa first be u.nrX,ir1!rr""rr"
Problem 2 - 53
A differential manometer shown is measuring the difference in pressure two
- Po=90kPa
w -ve-_e-_ ,*ewvs

water pipes. The indicating liquid is mercury (specific gravity = 13.6\,lu is


675

ethyl akohol mm, L,r is 225 mm, and 7r,,2-is 300 mm' What is the pressure differential
p = 773.3 kglm3 60m between the two PiPes.
Ans:89.3ZkPa

p = 899.6 kg/m3
oil : . lom
water
p = 979 kglm3 5m
glycerin
p = 1236 kglm3 5m
CHAPTER T\YO
7Z Principles of Hydrostatics FLUID MECHANICS
& lflDnAUUCs
Problem 2 - 54
A force of 450 N is exerted
on-rever AB as shown. The

5;:j""1ff1"1j::*:
acts on the larger piston, :XTi::1i;;;s-;;;*"ter
ii *,e .,otui";;i""""
or
end B is connected t
60 mm what rorce
i ""JJ;?il, #d,jfi::t Chapter 3
Ans:15.83

460 N
Total Hydrostatic Force
on Surfaces

TOTAL HYDROSTATIC FORCE ON PLANE SURFACES


lf tlre pressure over a plane area is uniform, as in the case of a horizontal
iurface submerged in a liquid or a plane surface inside a gas chamber, the
hrtal hydrostatic force (or total pressure) is given by:

F=pA Eq.3-1
t Problem 2 - Ss
where p is the uniform pressure and A is the area.
An open tube open tube is attached
I to a tank as shown. If water rises to a
height of 800 mm in the tube
l what are the prr
water? Neglect."pillr.y-;if*;ffi" Pa and pB of the air above tn the case of an iriclined or vertical plane submerged in a liquid, the total
,]
ffiPressures pressure can be found by the following formula:
I
Ans: pe = 3.92kpa; pa = 4.90
kpa

-l-
I

E
E
o
o

of gravity, cg

of.pressure, cp

Figure 3 - 1: Forces on an inclined plane


T
CHAPTER THREE CHAPTER THREE
74 Total Hydrostatic Force on Surfaces
FLUID MECHANICS FGUID MECHANICS
Total Hydrostatic Force on Surfaces
75
& HYDRAULICS i Hvnnnultcs
Consider the plane surface shown inclined at ah angle
0 with the horizontal.l IOCATI0N oF F (Yp):
To get the total force F, consider a differential eleme-nt of force .about s, (the intersection of the
of area ,rA. ;iltil: llt lltgrrrc 3 - 1, taking moment_
element is horizontar the pressure is uniform over this
area, then; plane area and the liquid surface),
;uulrriilirrtion bf the
dF=pdA t'n"=
where p = ylx tvar
p=y11sin0 where dF=yysin0dA
p=ysin0 A/
dF=yysinOdA
y sin0 Ay U, = U rinedA)
!V0
l'=r"i^e tvaa
From calculus,
I'n: = o, lsin0Aygtr=Ysin0
Iy2d A

F=ysin0 Ay
F=y(isin0)A
From calculus, p , dA =Is (moment of inertia about S)

AYYo=ls
From the figo.e, /sin0= f,
Then,

By transfer formula of moment of inertia:


since y f, is the unit pressure at the centroid of the prane
may also be expressed as:
area, p,s, theformura Is=[r+ rtlz
I. + Al2
Y? Ay

Eq. 3 - 2 is convenient to use if the plane is submerged


in a single liquid and
without gage pressure at the surface of the liquid. However,
if ,,re ptane is
under layers different liquids or if the gugu p."rr*e at the
:."bT:t9"9 _of
liquid surface is not zero, Eq.3 - 3 is easier to appry. sr" p?"BLi.
i - rs. Since y, = | + e, fromFigure 3 - 1, then

see Table 3 - 1 in Page76 for the properties of common plane sections.


fl"r
CHAPTER THREE
76 Total Hydrostatic Force on Surfaces FLU'D MIEHANICS CHAPTER THREE aa
& Total Hydrostatic Force on Surfaces ' '
I
TABLE 3 - 1: propefties of Common plane
sections
[Ialf ellipse Quarter ellipse

Area = 1/z rnb


4b
. Area = le rab
v'= gtr 4n
r'.=;- 4b
l-- olz
-.1-- 6p --r1 11,= ^31f
i

a+b
k- o_J l"='ttlo"
,1
1""=o.l"l
51f

I r: = -T- y. = h/3 Area= bd '8 abl . mtbj - nbn3


-16"1,6
i
, - bd3 db3 tba3
'r= -l-
Area = Vzb h ,sy_ Ir, = 0.055ab3 lrn= 0.055bar
l_
k=# u=#
t,
r
t bd3 ., dbl Sector of a circle Parabolic segment
l1

's,---uuT tw= AO-


I
al
I
i

t'
i

t,
h
t

l
Area = 1/z rt (20) = 7z 6 ' Aruu= ?bl,
x.=
2 rsin9 3
"30- 2,
x,=
'58-11 y,= 3,
-o
l,= -f,.(O-- %sin2e)
Area=t/ezr2; xr=y" ,4
Afea- fir2 = 1/4 z D2
I-= 2 2
t*=t*=+=+ fir'
n
-.4 '15"7 lrb, 1,,= bh,
Ir=1,= tr=
"L6 4
(0+t/zsn2O)
-
Ir,=1r=0.055/
Spandrel Segment of arc

T h ,s)
l_
k_o-l
Area=t/zn12; y,= Area= rab Length of arc = r(29) = 2rO
. rab3 1
x,=,-r siurt9
rf4 l9r-
-4 Area = ---:- b/r
Ir=1n=
*8 n +-L 0

Is, = 0.1't ,a , -
ttba3 )i.=
1 tt.=
rt +'L \Alhen 0 = 90' (semicircle)
tsy=
-Z- ' n+2 "' 4n+2
h
2r
Y-=
-b: - -T
7a
.v CHAPTER THREE
FLUID MlcHANlcs CHAPTER THREE ao
Total Hydrostatic Force on Surfaces
& Total Hydrostatic Force on Surfaces ' 7
TOTAL HYDROSTATIC FORCE ON
CURVED SURFACES
CASE I: FLUrD iS ABOVETHE CURVED SURFACE.

Whcrc:
1 A - vertical projection of submerged
r pressure at the centroid of A
curve (plane area)
[,Ls

Note: The procedure used in solving Fx is the same are that presented in Page 73.

Vertical projection iltrE III: FLUID BELOW AND ABOVE THE CURVED SUR'FACE
of the cunred surface

CASE II: FLUID Is BELOW THE CURVED SURFACE

l-
--IIIE|IEFF==

CHAPTER THREE
80 Total Hydrostatic Force on Surfaces
FLUID MECHANICS FLUID MECHANICS CHAPTER THREE ^ t
& HYDRAULICS * HYDRAULICS Total Hydrostatic Force on Surfaces U I

DAMS
Dams are structures that brock the flow of a river,
stream, or other waterway.
Some dams divert the flow of river water into
a pipeline, canal, or channel,
others raise the lever of inrand waterways to make'tt,e.r.r,arrig;ie
by ships
and barges. Many dams harness- theenergy of falling
power' Dams also hold water for drinking and
*ut".io g"'r-r"rute electric
crJp irrigatiJn, urra prorriae
flood control.

PURPOSE OFA DAM


Dams are built for the following purposes:
1. Irrigation and drinking.*uter-
2. Power supply (hydroelectric)
' 3. Navigation B 0at
leaurt
lork
4. Flood control
5. Multi purposes Flgure 3 - 3: Boat Passlng through Canal Lock. Canal locks are a series of gates designed
t9 allow a boat or ship to pass from one level of water to another. Here, after a boat has
entered the lock and all gates are secured, the downstream sluices open and water.flows
through them. When the water level is equal on either side of the downstream gate, water
stops flowing through the sluices; the downstream gate opens, and the boat continues on at

To ttanrmirrion linrt

TYPES OF DAMS
1. Graoity ilams :use only the force of gravity to resist water pressure-
that is, they hold back the water by the sheer force of their weight
pushing downward. To do this, gravity dams must consist of a mass
so heavy that the water in a reservoir cannot push the dam
downstream or tip it over. They are much thicker at the base than the
top - a shape that reflects the distribution of the forces of the water
against the dam. As waiter becomes deeper, it exerts more horizontal
pressure on the dam. Gravity dams are relatively thin near the surface
of the reservoir, where the water pressure is light. A thick base
enables the dam to 44withstand the more intense water Pressure at
Figure 3 - 2: Section of a dam used for hydroelectric the bottom of the reservoir.
'-"-

CHAPTER THREE
82 Total Hydrostatic Force on Surfaces
FLUID MECHANICS CHAPTER THREE
Total Hydrostatic Force on Surfaces
83
& HYDIIAULICS

1 A lutttrcss ilam consists of a wall, or face, supported by several


lrrrltrt,sscs on the downstream side. The vast majority of buttress
rlnnn irrc macle of concrete that is reinforced with steel, Buttresses are
lypicirlly spaced across the dam site every 6 to 30 m (20 to L00 ft)'
.1.,1r,'.,.ling upon the size and design of the dam' Buttress dams are
s,,irrt.timei .iu"d hollo* dams because the buttresses do not form a
nolid wall stretching across a river valley'

Figure 3 - 4: Gravity dam

2. An embarukment dam is a gravity dam formed out


of loose rock,
earth, or a combination o? theie materials. The
upstream and
downstream slopes of embankment dams are flatter
than those of
concrete gravi\r dams. In essence, they more closely
match the
natural slope of a pile of rocks or earth.

3' Arch clarus areconcrete or masonry strucfures that


curve upstream into
a reservoir, stretching from one wail of a river
canyon to the other. This
design, based on the same principles as the architectural
arch and vaurt, Figure 3 - 6: Buttress dam
transfers some water pressure onto the walrs of
the canyon. ar"i au*,
.T"it_u a relatively nallow river canyon with solid
roct waUs capaUte
of withstanding a significant amouni of horizontal
thrust. These dams
do not need to be as massive as gravity dams
becausu trr" r"utt,
carry part of the pressure exerted by the reservoir. "unyor,

Figure 3 - 7: MultiPle arch dam


Figure 3 - 5: Arch dam
r r{-FircffiE-

ii'

CHAPTER THREE
84 Total Hydrostatic Force on Surfaces FTUID MICHANICS CHAPTERTHREE ftF
I
&l Total Hydrostatic Force on Surfaces O)
ANArY$rs oF GRAVTry DAM
i
l. A dam is subjected to hydrostatic forces A, VcrLical forces
due to water which is raised on
l, upstream side' These forces
cause thu Ju^ to slide t o.iror.,tuuy
1, Weight of the dam
Wr=f"Vr) W2=f,Vz) Wz=!"Vz
*lf:::"::1^-:^".:lyl it "uo"i-ii*dl_*t
on
tendencies are resisted by fricuon "u* edge or roe. rh
on;";;;;;T,:X.*T 2, Weight of water in the upstream side (if any)
#r;"^i
opposit" io ih" o,",tu,,,ing moment.
Ws=YVa
:i:,:T:.-:::::::,i11il
fmay also be prevented from sliding
Uy t"yi"g f;b"r".
rhe 3. Weight or permanent structures on the dam
4, Hydrostatic Uplift
Upstream Side
Ur=f Vut
Downstream Side Uz=! Vuz

(Tailwater)
Headwater Il. l{orizontal Force
1. Total Hydrostatic Force acting at the vertical projection
of the submerged portion of the dam,
Vertical p=yhA
I Projection of
I
I
the jubrleqed- 2. Wind Pressure
I

!l face ofl-dam J. Wave Action


h
I 4. Floating Bodies
5. Earthquake Load
I
I
Ill. Solve for the Reaction
t,i

I
A. Vertical Reaction, R,
1m Heel
Ry = XF,,
l Rv=Wt+Wz+Ws+Wa-Ut-Uz
Uplift Pressure
Diagram ---> B. Horizontal Reaction, R,
R' = EFr'
R'=P
Zz IV. Moment about the Toe
&i A. Righting Moment, RM (rotntion toroards tlrc upstream side)
Figufe 3 - 8: Typicar section of gravity
a dam showing the possibre forces KM=W, h + Wz xz+Vrls xe + We xt
acting
Steps of Solution
B, Overturning Moment, OM (rotation towards the downstream sidc)
to Figrr-r:
-,Sr.forpurposes of illustratiory qrr qooq, OM=P!+UtztrUzzz
made l"Y*:"
I:,1 B
in the shape of the'uptift prdr"; - --*-", an assunption was
;;;u*.
I. Consider 1 unir (1 m) Iength of dam V. Location of R, (; )
lf,erpenaicular to rhe sketch)
II. Determine all the forces acting:
KM_OM Eq.3 - 11
R,
CHAPTER THREE
86 Total Hydrostatic Force on Surfaces
CFIAPTER THREE
Total Hydrostatic Force on Surfaces 87
where: Ir.

y = unit weight of-water 9.g1 n, * 6R..e


= kN/ms (or 1000 kg/mr) lr r*
yc = unit weight of concrete I -!-B'
T, = 2.4y (usually taken as 23.5 kN/m)

Factors of Safety
Factor of safety against sliding, FS5:

Ure (+; to get the stress at point where R, is nearest. In the diagram
tbove, use (+1 to get 4r and (-) to get qH. A negative stress indicates
stress and a positive stress indicates tensile stress:

Factor of safety against overturning, Fse: loll cannot carry any tensile stress, the result of Eq. 3 - L4 is invalid if the
lo positive. This will happen rt e> B/6' Should this happen, Eq.3 - 15
be used.

p = coefficient of friction between > 8/6


the base of the dam and the foundation when e

Foundation pressure
7 =a/3'
t- Bl3 I BIS t, eB
r Middle Third I
For e 3 8/6 a-37
I

From combined axial and bending


stress formula: R, = 1/z(a)(q,)(1)
Rv = r/z{37)qa
o=-! M,
, A-* I I
P=R, Heel
A=B('1.)=B Qx
M= Rre
1(B)3
, -
12
c=82
R, (R, e)(B / 2)
,B
u___ + ------=-
85 /12

d&i-
-l'l'

CHAPTER THREE
88 llUlo tvtpcsANtcs CHAPTER THREE
Total Hydrostatic Force on Surfaces
I HYDRAULICS fotal Hydrostatic Force on Surfaces 89
BUOYANCY
where:
y = unit weight of the fluid
ARCHIMEDES' PRINCIPLE yD = volume displaced. Volume of the body below the liquid surface
A principle discovered by the Greek scientist
Archimedes trrat states that ,,,
body innrcrsed in a fruid is acted acting and npply conditions of static
"p-;i;;;-;';iiora y*r, (buoyant To mlae problenrs in buoynncy; identifu the forces
roeigli of the di splnied force) equal to
fhtid,' . aquilibriuru:
IFs=0
This principle, also known as the_lazo XFv=0
of lrydrostatics,
, applies to both
and submerged bodies, and to all
fluids.' " 2M=0

For homogeneous solid body of volume V "floanng" in a homogeneous fluid at


fost:

yr= rp.g..oLfod,y,
sP. 8r. ot llquld
v=To"dv t/ Eq.z_L7
Y [quitl

If the body of height H has a constant horizontal cross-sectional area such as


vertical cylinders, blocks, etc.:

p= sP'Er.ofbody Yuoay
H= , Eq,3 - 18
Figure 3 - 9: Forces acting on a submerged
body

BF=Fvz-Fvt
= y(Votz) - 7(Volr) If the body is of uniform vertical cross-sectional area A, the area submerged A'
BF=y(VoI2-Volr) is:

',,,,,. nqq"rtn== 91r; \aoav


o
,,:'
",' ,.,..,.,.i;l;
l:':l:,]:.:-sp.gr.ofliquidYuquia
Eq.3-19
rr,--
CHAPTER THREE
90
I

Total Hydrostatic Force on Surfaces MICHANICS CHAPTERTHREE ol


Total Hydrostatic Force on Surfaces ' t
STATICAL STABIUry OF FLOATING
BODIES
A floating body is acted upon by two equal opposing
forces. These are;
weight w (acting it its ce.,t"r ,i;;"ri;,J-;;?'i,,
9.9.y''
(acting at the center of buoyancy that is loc?ed
o""ir^, ,"r."
at the center of gravity of
displaced liquid).

when these forces are coninear as shown in


Figure 3 - 10 (a), it floats in
upright position' Ho*"ygl, when the body
t,ts due to wind or wave acti
the,c.enter of buoyancy.:lifts ,o it
p.linon u, shown in Figure 3 - 10
""-
ronger' corrinear,,produce,
il,i,'T#:^i:::1,:T:L. 1: ""
ll(x). The bodv wil not overtum if this coupre;;:;il&;:i:;T, ";il;i;q;l
its originar position as shown in Figure
situation is as shown in Figure 3 _ 10
s i ro (b), and win overturn if
[c).
The point of intersection between the - 10 (c): Unstable Position
axis of the body and. the line of action Figure 3
calted the rn etacenter.-ih" dirtur,ce from
ot
(M) i:"_r:^I.:
llr: to :
the center of
the metacenter
(c) or *re ilody is ca,ed ,n" *"r"""riilr'JTlf,Tl Figure 3 - lO: Forces on a floating body
Tu:r?
(MG)' lt can be ,"".r ihut a uoay is stable it tvt rsabove G us
- 10 (b), and unstabre if M is terow c ur rno-.;; sho*r, in Figure B
Fd;"1"-"i'o o If M
coincides with G, the body is said to
be just stable. IIOHTING MOMENT AND OVERTURNING MOMENT

RMoTOM=WI*) ,8q,3-20

ILEMENTS OF A FLOATING BODY:


W = weight of the body
BF = buoyant force (always equal to Wfor a floating body)
G = center of gravity of the body
Ilo = center of buoyancy in the upright position
(cenkoid of the disPlaced liquid)
Ilo' = center of buoyancy in the tilted position
Vp = volume displaced
M = metacenter, the point of intersection between the line of action
of the buoyant force and the axis of the body
c = center of gravity of the wedges (immersion and emersion)
s = horizontal distance between the cg's of the wedges

Figure 3 - u = volume of the wedge of immersion


10 (a): Upright position
l

Figure 3 - 10 (b): Stable position 0 = angle of tilting


Mpo = distance from M to Bo
GBo = diqtance from G to Bo
MG = metacentric height, distance from M to G
CHAPTER THREE
92 Total Hydrostatic Force on Surfaces CHAPTER THREE
Total Hydrostatic Force on surfaces Gt2
7)

due to shifting of BF = moment due to shifting of wedge


Use (-) if G is above Bo
Use (+) if G is below Bo
5F 1r1 ' 1' 1t1
Bl'-,t Vo

Note: M is always above


f'-yu
Bo
:-MBo sinO

lVoMBosin0=yos
VALUE OF MBO
The stabirity of the body depends
on the arl0unt of the righting momr
which in turn is der:endent o, t."
*"iu.u"oi" heightMG. wh;;; body tt
the center of buovancv shifts
to , ;;;;ri,t" n (Bo). This shifting urro
the wedge to rhift io u ,,"* position o. The
the buoyant'' force BF(z) is mustLquario
"aur
moment due to tlie shifting
*o*"r,t due to wedge shift F(s). Ilmll vALUE oF MBo
F€] tmall values of 0, (0 ^, 0 or 0 = 0):

Pitching

ee
(B/2) tan e

Figure 3 - 11: Rectangular body


Waterline Section
( irnsider a body in the shape of a rectangular parallelepiped length L as
llrt'rwn in Figure 3 - 11;

Volume of wedge, a = Yz(B / 2)l(B / 2) tan 0)L


Volume of wedge, p = f LBz tan 0

Forsmallvalues of 0,so !B
94 CHAPTER THREE GHAPTER THREE oq
Total Hydrostatic Force on Surfaces Totat Hydrostatic Force on Surfaces ' r

MB.= " SECTION


Vp sin0

*r"= *LBz tano*ZB


But for small values of 0, sin 0 * tan 0
Vp sin0

MB"=
*ts3
Jt-- (B/2) sec e
VD

But fi LB3 is the moment of inertia of the waterline section; I

}-(B/2)
tan e
-t_
Note: This formula can be applied to any section,

since the metacentric height &-rG is dependent with MBo, Centroid of wedge
the stability
floating body therefore depends on the moment of inertia [1q, 3 - 22,
of the watt
section. It can also be seen that the body is more stabre in pitching
rolling because the moment of inertia in pitcning is greater
than
tnan trrat in rolllt
irr,-dk
Vn- BDL where L is the length perpendicular to the figure
rt * t/z(B/2)l(B/2) tan0lL

MOMENT
1r- fiLBztan0
('entroid of triangle, f
The righting or overturning moment on a floating body
is: xt + xz + xz
From geome try,i =
3
-- 0+ (B/2)sec9+ (B/2)cos0

B ('
*.ore)= r [r+cos-'?e]
t = A[..oro-'""",, 5[ coso )
s - n (t+.or'e)
2 6 [ cos0 )
n (t+.or'e')
s=-l-l
3[ cos0 )
,ar
'1

CHAPTER THREE
96
I

Total Hydrostatic Force on Surfaces FLUID MECHANICS CHAPTER THREE


& HYDRAULICS
FLUID MECHANICS
& HYDRAULICS Total Hydrostatic Force on Surfaces 97
(lonsider a pipe of diameter D and
MBo= thlckness f be subjected to a net Pressure
(BDt)sin0 p, 'l'o determine the tangential stress in
the pipe wall, let us cut a section of length
LB3 sin0 1+cos20
r nlong the diameter. The forces acting on Projection of
MB, = U-;;e '-;;;o- this section are the total pressure F due to
curved area
BDL sin 0
the internal pressure and this is to be

MBo= 82 1+cos2o rcsisted by T which is the total stress of


t.
24D cor2 O the pipe wall.

I
I ias,= Ll,-+-.r)
24D\"r"2,'-1
Applying equilibrium condition;
[EFa = 0]
rf o2 F =27
t'
*r"=#(secz0+1; butsec20=1+14n26 F=pA=pDs
T = Sr A*"n
I T=51(sxf)
,r
I
pDs=2x[Sr(sxf)]

nD
Tangential stress, Sr= Eq.3'26
fr

To determine the longitudinal stress, let us


cut the cylinder across its length as shown.
[XFa = 0]
F=T
STRESS ON THIN-WALLED r=pA
PRESSURE VESSELS F=P +D'
THIN.WAIIED CYTIN DRICAT TANK T = Sr A*"rr
A tank or pipe carrying a fluid or gas Anat= nDt
under a pressure is subjected to tensile
resisi bursting, aurJrop"J-u*oss T = SrnDt
i:[:lJ*rh rongitudinar and kansverse

p 1D'= StrDt

,,.,., : .r ., Longitudinalstress, ^pD


Sr= * Eq,3-27
:a

= internalpressure - external pressure Eq. 3 - 28


CHAPTER THREE CHAPTER THREE
98 Total Hydrostatic Force on Surfaces
FLUID MECHANICS
& HYDRAULICS
FLUTD MECHANICS
& HYDRAULICS Total Hydrostatic Force on Surfaces 99
SPHERICAL SHELL
If a spherical tank of diameter D and thickness f contains gas under a pressure Solved Problems
of p, the stress at the wall can be expressed as:
Probtem 3 - 1
A vertical rectangular plane of height d and base b is submerged in a liquid
with its top edge at the liquid surface. Determine the total force F acting on
one side and its location from the liquid surface.

Solution
p=yhA
watlstress, s=+
4t
Eq.3-29 n=a/z
A=bd
F = y(d/z)(bd)
F=r/zybilS
SPACING OF HOOPS OFA WOOD STAVE PIPE
t.o
e=
Ay
I =h =d/2
*ua'
(bd)(d / 2)
e=d/6

yr=i +e
yo= d/2+ d/5
Vp= 2iV3

Using the pressure diagram:


sPacrn& 5^= -:k:-
25, A, F = Volume of pressure diagram
pu
,: Eq.,3 - 30
F=%\A@)(b)=r/rYb iP

where: The location of F is at the centroid of the pressure diagram.


Sr = allowable tensile stress of the hoop
A,r = cross-sectional area of the hoop
For rectangular surface (inclined or vertical) submerged in a fluid with top edge
flushed on the liquid surface, the center of pressure from the bottom is U3 of its
p = internal pressure in the pipe height.
D = diameter of the pipe
I t oo ;1,:il;5,:H:[ Force on surraces
FLUID MECHANICS FLUID MECHANICS CHAPTER THREE
lol
& HYDRAULICS & HYDRAULICS Total Hydrostatic Force on Surfaces
Problem 3 - 2
Sotution
A -vertical- triangular surface of height d and
horizontal base width b is p=yhA
submerged in a liquid with its vertex a1 the liquia
surface. Determine the total
force F acting on one side and its location from F = y(r)(nz2)
the liquid surface.
F=n.ff
Solution
r=via I,
(, = ---.3-
n = *a Ai
. t--4
A= l/zbd +;tLI
. e: 7
=r14
F=yrta"vzUa (nr')(r)
I

F= Yr=r+e
lrb* 'yu=r+rf4
Pressure diagram
yo= 5r/4 (cylindrical wedge)
t"6
Ay Using the pressure diagram for this case is quiet complicated. With the
shape showry its volume can be computed by integration. Hence, pressure
y =i =Zd/g
diagram is easy to use only if the area is rectangular, with one side horizontal.
1 UU
r:3
TE
r--
Gbd)(zd /3)
e = d/12 Problem 3 - 4
pressure diagram
( (pyramid) A vertical rectangular gate L.5 m wide and 3 m high is submerged in water
yo= i + e with its top edge 2 m below the water surface. Find the total pressure acting
on one side of the gate and its location from the bottom.
Yo= ?d+ d112=s4tr4

Using the pressure diagram: Solution


F = Volume of pressure diagram p=yi.A
F= *An","xheight n'=1.5+2=3,5m
F= +@xyd)(d)=
*yaa, F = e.81(3.5)[(1.5X3)]
F = 154.51 kN
F is located at the centroid of the diagram,
which is % of the altitude
from the base.
I"6
Ay
Problem 3 - 3
A vertical circular gate or radius r is submerged in -0.21.4m
a riquid with its top edged (1.5 x 3)(3,5)
on the liquid surface. Determine th'e magnitude and
ftusfga location of the Y=1.5-e
total force acting on one side of the gate.
y=1.5-0.274
A =l.2f,6nn
Ii
oz
I

CHAPTER THREE
l

I ;m,tIff':H.'L Force on Surraces


FLUID MECHANTCS
& HYDRAULICS
FLUID MECHANICS
& HYDRAULICS Total Hydrostatic Force on Surfaces r03
,i
l

Using the pressure diagram: Solution


F = Volume of pressure diagram
p=yiA
,=(u?,s)osr i =z+ $(e)
F = 15"75y n =3m= y
F = 1s.75(e.81) F = [e.81(0.82)](3)[%(1.sX3) ]
F = 154.51 kN F = 54.3 kN

t" *(r.sxs)3
Ay t+(1.5X3)l(3)
e = 0.\67 rn
yr=i +e
y, = 3.167 m from the oil surface

3y 2t Probtem 3 - 6 (CE Board May 1994)


Pressure diagram
(trapezoidal prism) A vertical rectangular plate is submerged half in oil (sp. gr. = 0.8) and half in
Location of F:
water such that its top edge is flushed with the oil surface. What is the ratio of
' At = 2y(3) = 61 the force exerted by water acting on the lower haU to that by oil acting on the
Az=1/z(3y)(3) = a.5y upper half?
A=Ar+Az=10.5,
[Ay =zayl
Solution
1.0.5y y = 5y(1.s) + a.5y(1) Force on upper haU:
V = 7.286 m (much complicated to gef than using the formula) Fo=yoi A
Fo = (T. x 0.8)(d / +)lb(d / 2)l
Fo = 0.1y,ub iP
Problem 3 - 5
Force on lower half:
A vertical triangular gate with top base horizontal and 1.5 wide is 3 m high. It Fw= psg2x A
is submerged in oil having sp. s. of 0.82 with its top base submergeJ to a *
Pcg:- = To ho Tw hu
depth of 2 m. Determine the magnitude and location of the total hyirostatic
pressure acting on one side of the gate. P,9= (T.x0,8)(d/2) + Y,"(d/4)
Pcgz= 0.55 h, d
F67= (0.65 y* d)fb(d / 2)l
Fw= 0.325T*b dz

Fw
Ratio =
Fo

o.g25t*bl2
Ratio= =g.x
0.1y,ubd'
r
I 04 f#,t;ffi:[ff Force on surraces
FLUID MECHANICS
& HYDRAULICS
FLUID MECHANICS
6. HYDRAULICS
CHAPTER THREE
Total Hydrostatic Force on Surfaces t05
Problem 3 - 7 (CE Board May 1994)
Problem 3 - 8 (CE Board May 1992)
A vertical circular gate in a tunner g m in diameter has oil
- (rp. gr. 0.g) on one A closed cylindrical tank 2 m in diameter and 8 m deep with axis vertical
side and air on the other side. If oil is r.2 m above tt,e inrreit
and the air contains 5 m deep of oil (sp. gr. = 0.8). The air above the liquid surface has a
pressure is 40 kPa, where will a single support be rocated
(above the invert of pressure of 0.8 kg/cm2. Determine the total normal force in kg acting on the
the tunnel) to hold the gate in position?
wall at.its location from the bottom of the tank.
Solution Solution

Oil; s 1 0,Q

Air)P=agP3

+,- 8mA
4m
I

Foit=loli. A
r"i_J_
F,l = (9.81 x 0.S0)(S) Fr=P"nA
><
f (8),
par = 0.8kg/ crfi = 8,000 kg/ rr:,2
F ir = 3,t56 kN
F1 = 8,000(2n x 2) = 349ggn Ut
,=" h=5+1=7m
Ay
Fz= PreA
e- +8\4
n'=' =0.5m p.s = (1000 x 0.8X3) + 8,000
f (8)'(8) Pcs= 10,400k9/m'
z=4-e=3.5m Fz= 70,400(2n x 6) = "124,800nkg
Solve for e:
Fair=PairAr=40x
f;(8), Fz=y"i A
Fop = 2,01L kN
124,800n= (1000 x 0.8) h (2n x 6)
The support must be located at point O where the moment due n = v =13m
to Fur
and F,ir is zero. Since F4 ) Fri,, O must be below Foir.
[XMo = 0]
r I g1zn11oy3

Foit(z-y)=F,i,(4-y) Av (2lt x 6)(13)


(3,156X3.5 - y) = 2,011.(4 - y) e = 0.23077 m
1.569(3.5 -y)=4-y !/2=3-e=2.77m
5.493-1,.569y=4-y
y =2.62m F = Fr + Fz = 154800r kg ) Total normal force
,lll
CHAPTER THREE
rili
I 06 ;I*,'Iil':H:,: Force on Surraces FLUID MECHANICS
& HYDRAULICS Total Hydrostatic Force on Surfaces l07
[Jllr Fy=Fr1,l1+Fzyz Solution
Hl l

lrSe,SOOn; y = (32,000r)(n + 024,800n) (2.7V


[E Mr.*g" = 0l
ril
Iit
= 3.53 m ) Location of F from the bottom F z = 40(7)
tlli
flli
Using the pressure diagram: E =yh A=9.81.h (1X1.5)
hl
tit'. F =14.715i
Ill
Iti
ifll
r

i
I^ -T--
$t
e= -L
Av
where / = ft z
frl
-L

IE
fll
8 g=
|1r.s11r13 1

I 6
(1.5 x 1)h 12h

Htil I

i i l-'; z=0.5
- +e=0.5*
-
1

800(6) = 4800 8000


-tzh
m; /r\

Pressure Diagram 14.715h [0.5* '=l=10


fl, \ 12h)
lli'lr
{t Pr = 8000(8X2n) = 128,000n kg 0.5h +0.08333=2,718
I Pz= 1/z(4,800)(5)(Zr) = 28,800a kg n =S.zZ*= [ +0.5=5.Tl1ea1 ) critical water dePth

{
i P.= P1 + P2=156,800*kg ) Total normal force
Problem 3 - 10
lPy=hy1+Pzllzl is flushed
(155,800r) y = (128,000n)( ) + (2S,800r)(2) A vertical circular gate is submerged in a liquid so that its top edge
with the liquid suiface. Find thJ ratio of tolal force- acting on the lower
y = 3.53m ) Location of P from the bottom the
half to that icting on the upper half,

Problem 3 - 9 Solution
In the figure showry stop B will ' Ratio =
F2

break if the force on it reaches F1


40 kN. Find the critical water
depth. The length of the gate -
,b
Ratio = ^,
Yht Ar
perpendicular' to the sketch is
i

1.5 m At= Az

Ratio = b
h7

Ratio = =2.475
#,
I oB fL:i;;H?[:J: Force on Surraces
FLUID MECHANICS
& HYDRAULICS
FLUID MECHANICS
& HYDRAULICS TotatHydrostatic
CHAPTER THREE t ,it
Force'j"=';#;::: lOq
Problem 3 - 11 Solution
A 30 m long dam retains 9 m of L=30m
water as shown in the figure. Find
r=y nA
the total resultant force acting on the
dam and the location of the center of
i =3.5 + 2/3
pressure from the bottom. i = 4.157 rn
A=1/z(7)(2.51)
A = 1,305 m2
P= (9810 x 0.$)(a.157)(1.305)
F = 44,277 N
Solution E = tM.277kN

f=yhe
f = e.81(4.5)[(30)(10.3e2)]
F = 13,763 kN
Problem 3 - 13
L=30m
I. An inclined, circular
6
gatewith water on one
AV
side is shown in the
fi1ao;1ro.sez;3
.h = 4;5 figure. Determine the
e= total resultant force
(30 x 10.392)(4.5 / sin 60")
acting on the gate.
e =L.732m

y = 1/z(10.392) - 1..73;2
y = 3.464m

y= +(10.3s2',)=3.464m

Problem 3 - L2
Solution
The isosceles triangle gate shown
in the figure is hinged at A and F=vhA
weighs 1500 N. What is the total
- i =2+o.5sin5oo
hydrostatic force acting on one side i =2.433
of the gate in kiloNewton? F =e.87(2.433)t GF
F = 18.746 kN
I r o ;:lfff;il:l:ffForceonsurfaces
FLUID MECHANICS
& HYDRAULICS
FLUID MECHANICS
& HYDRAULICS
CHAPTER THREE
Total Hydrostatic Force on Surfaces ttl
Problem 3 - 14 I.6-
(b)
#(r.sXs.o)3
The gate in the figure,shown is L.5 m wide, hinged at point A, and rests Ay (7.5x3.6)(7.21)
against a smooth wall at B. Compute (a) the total force on the gate due to
seawater, (D) the r€action at B, and (c) the reaction at hinge A. Neglect the e = 0.1.5 m
weight of the gate. r=1.8-0.15
x=1.65m
[XM^ = 91

F(x)-Rr(2)=g
218.25(1.65) = 2P,
RB = 180 kN

(c) [EFg = 0]
' Ra,+Fsin0-RB=Q
Rar, = 180 - 278.25 sin 33.69o
Rei, = 58.94 kN
tEf,=01
Rau-Pcose=0
Rm=218.25 cos 33.69o
Ra, = 181'5 kN

181.6)2 + (s8.94)2

Solution

Problem 3 - 15
*-32+22 Determine the magnitude
d=3.6m and location of the total
hydrostatic force acting on
tan9 = 2/3 the?mx4mgateshown
0 = 33.69' in the figure
i
" .sin e
_4
;li
'u=- sin 33.59o
i =7.2t m

(a) r=yia
F = (e.81 x 1.03)(a)[(1.sX3.6)]
F = 218.25 kN
I l2 CHAPTER THREE
Total Hydrostatic Force on Surfaces
FLUID MECHANICS
& HYDRAULTCS
FLUID MECHANICS
& HYDRAULICS
CHAPTER THREE
Total Hydrostatic Force on Surfaces t t3
Solution
Problem 3 - 16 (CE November 1997)
1

Determine the magnitude of


m the force on the inclined gate
1.5 m by 0.5 m shown in the
1.5 m Water Figure 00L. The tank of
water is completely closed
Figure 001
and the pressure gage dt the
bottom of the tank reads
l

m
90,000 N/m,. Use %800
N/cu. m. for water.
;rr1,;1t1qil,::t!\./f
'v
, )q
ll'
.l

i'
F = prrA

P,s= zyb + p
p,s= (9.8tx1.26)(3) + (9.81)(1.5) + (9.81x0.80)(1) + sz
p,r= 97.645kPa
Solution
F=97.645(2x4)
F = 733.15 kN
F = pr"A
Solving for e: Pz - Pcg= \h
for h and
90000 - p,s= 9,800(2,55)
Solve ! :
p,g= 6403APa
F=yh a F = 54030 (0.5 x 1.5)
733.1,6= (9.81x7.26)n p* +1 F = 48,022.5 N
i =7.41,4m
i = n /sin 60. =7.414/ sin 5o'
7 = 8.551m

o=',
Ay =(2 x a)(8.s61)
#(z)(+)'
Pz = 90,000 Pa

e = 0.156 m
Problem 3 - L7
z=2-e=1.844m The gate shown in the figure is hinged at A and rests on a smooth floor at B.
The gate is 3 m square and oil of having sp. gr. of 0.82 stands to a height of 1.5
Therefore, F is located 1.8rM from the bottom of the gate.
m above the hinge A. The air above the oil surface is under a pressure of 7 kPa
above aknosphere. If the gate weighs 5 kN, determine the vertical force F
required to open it.
T'

t l4 CHAPTER THREE
Total Hydrostatic Force on Surfaces
FLUID MECHANICS
& HYDRAULICS
FLUID MECHANICS
& HYDRAULICS
CHAPTER THREE
Total Hydrostatic Force on Surfaces I l5
Problem 3 - 18 (CE Board)
Iron pins 20 mm in diameter are used for supporting flashboards at the crest
of masonry dams. Tests show that the yield point of iron to be 310 MPa
(extreme fiber stress). Neglecting the dynarnic effect-of watqr on flashboards
and assuming static conditions, what is the proper spacing, S, of the iron pins,
so that the flashboards 500 mm high will yield when water flows 150 mm deep
over the top of the flashboards.

Sotution

I
Solution
P = prrA
,iN pcg= pair + loJlo
P,s = 7 + 9.81(0.82)(2.55)
lr
pcg=27.59kPa
l,'
P =27.5e tG)G)l
P = 248.34 kN

p=yhA
248.34 = (9.81x0.82)E 1a, a;
Ir

h =3'.43m
h 3.43
sin 45o sin 45o
3 sin 45' = 2.12 m Moment capacity of one iron pin (20 mm @):
= 4.85 m
lFn= Mc/4
t- M(4)
g= --!- = #(e)(s)' 310= ':'\2/'
Ai (3 x 3)(4.85) *(20)*
e = 0.155 m M=243,473.43 N-mm
x=1.5+e M= 0.24347kN-m
x - 1.655 m
Moment caused by F (considering S m width of flashboard):
[El't^ = g 1 Mr;F xY
P(x) + W(1.06) - F(2.7\ = a
2.1,2F = 248.34(1.55s) + 5(1.05)
P=yhA whereA=0.6S
. F=L95.37kN F = 9.8L(0.45)t0.5 Sl
. I '
I IO
CHAPTERTHREE
Total Hydrostatic Force on Surfaces
FLUID MECHANICS
& HYDRAULICS
FLUID MECHANICS
& HYDRAULICS
CHAPTER THREE
Total Hydrostatic Force on Surfaces tl7
F = 2.6495 Problem 3 - 20
lt=0.3-e At 20'C, A in the figure reads 290 kPa absolute. The tank is 2 m wide
gage
t"o_ #(sxo.6)3 perpendicular to the figure. Assume atrnospheric pressure to be 1 bar. Sp. gr.
Ai (0.6 s)(0.4s) of mercury = 13.6. Determine the total pressure acting on side CD.
e = 0.067 m
y = 0.3 - 0.067 = 0.233 m -T-
Mr=Fry=M 1m
.1.
2.6495x0.233=0.24347
S=0.394m=394mm t
h

J
Problem 3 - 19
70 cm
The semi-circular gate shown
in Figure 28 is hinged at B. Gage A
Determine the force F required
to hold the gate in position.
Solution
Solving for ft:
, Solution %=Zyh+pnp
i = y =10-1.598 299 = (9.81. x 13.6)(0.70) + (9.81)h + 175
h=2.2m
i =, =8.302ft
f=yhe Total force on side CD: (Note: 1 bar = 100 kPa)
P = 62.4(8.302)flzrcg)21
P = 13,019.89Ibs pr=175-100
t"d 7=75kPa 1m
c--
p2= 9.81(2.9)
AY
p2= 28.M9kPa
I

I
Ig = 0'1098
i, = 0.1098(4)a Ft= pr(3.9)(2)
ls = 28'77 fta h-75(3.e)(2)
4fr.
-_
e
28_.11

(8.302) _L_
Fr = 585.kN
+n(4)2
z Fz= 1/zpz(2.9)(2)
', ,=0.1347ft Fz = 1/z(28. aae) (2.9) (2)
b = 1,.698 - 01347 = 1.5533 ft Fz = 82.5 kN 2m

lIMs = 0I F=h+Fz
P(b) = F(4) F = 657.5 kN
13,019.89(1.5 633) = P g7
F = 5088.5 lbs
I l8 fI*i;;I..lH:[ Force on surraces
FLIJID MECHANICS
& HYDRAULICS
FLUTD MECHANICS
& HYDRAULICS
CHAPTERTHREE
Total Hydrostatic Force on Surfaces It lg
t7

Problem 3 - 21 Solution
The funnel shown in the figure is p=yhA
full of water. The volume of the r = e.81(2.6X1.6 x1,.2)
upper part is 90 liters and the F = 48.97 kN
lower part is 74 liters. What is the
I.
force tending to push the plug e=4
out? AY
t.z0.q3 2.5m,
72
Afea = 100
(1.6x1,.2)(2.6)
e = 0.082 m
z=0.8-e
z=0.718m
= 460 cm2
T=Fxz
Solution T=48.97x0.718
since the plug area in contact with water is horizontal, the pressure all over T = 35.15 kN-m
it is uniform. The shape of the container does not affect the pressure on the
Plug'
Force=pxA Problem 3 - 23 (CE Board)
9,810(3)(#
Force = ) A cubical box, 1.5 m on each edge, has its base horizontal and is half-filled
Force = 1353.78 N with water. 'The remainder of the box is filled with air under a Sage Pre55ure
of 82 kPa. One of he vertical sides is hinged at the top and is free to swing
inward.' To what depth can the top of this box bei submerged in an open body
Problem 3 - 22 of fresh water without altowing any wafer to enter?
In the figure showrU the gate AB
Sotution
rotates about an axis through B.
The gate width.is 1.2 meters. A
torque I is applied to the shaft
through B. Determine the torqre T
to keep the gate closed.
;ill
l
i;i'
i

in,,'
li,
]ii

9.81(oJ5) 82 kPa
= 7,36 kPa
I 1.5mx1.5m
I

,]
l
tul
:-t
l
l,.r[,i-,,

lr
rl

l20 f#i;i:,:H:,: Force on surfaces


FLUID MECHANICS
& HYDRAUTICS
FLUID MECHANICS
& HYDTiAULICS
CHAPTERTHREE
Total Hydrostatic Force on
Ia
Surfaces t tL 1t
ll''l
rll
[E Mni^g" = 0] Problem 3 - 24
Fs (r) - Fr (0.75) - Fz (1.25) = s ) Eq. (1)
Find the magnitude and location of
rl Ft = P^rA
the force exerted by water on one
,l n = 821(1.sx1.5)l = 184.s kN
;l
side of the vdrtical annular disk
il shown. 4m
F, = Y2(7 .36) (0. 75X1. s)
,t R=1.5m
Fz = 4.14 kN I
II I

il 4=.yi A i_
il
ll Fs= e.81ii 11r.sy1r.sy1
',1
rl
Fs= 22.07i

,li x = 0.75 +'e


1.5(1.s)3
Solution
F=yhA
I-
g= -!- = 12 F = 9.81(4)[zr(1.5), - r(1)r]
il Ay l(1.5X1.5)lt F = 154.1 kN
0.1875
0=:
iL,

!l .h Location of F:
l,
x = o.z5+
0'1875 t*f(l.s)n -xQ)4
- Av-- n[(1.5)2
i1, h - (1)2](4)
ir In Equation (1): e = 0.203 m
I

zz.ozi (0.75 + vp= 4 + 0.203 = 4.203 m below the w.s.


ryP
h
) - 184.s(0.75 ) - 4.1,4(1,.2s) = o

16.55h + 4.138 - 1g9.g75 - 5.175 = 0


1,6.55 h =139.412 Probtem 3 - 25
h = S.42rn The gate in the figure shown
weighs 5 kN for each meter
h= h -0.75 normal to the paper. Its center of
h=7.67 m
gravity is 0.5 m from the left face
and 0.6 m above the lower face.
Find ,1 for the gate just to come
up to the vertical position.
, ,-t CHAPTER THREE CHAPTERTHREE I A?
Surfaces I Lr
FLUID MECHANICS
I ZZ Total Hydrostatic Force on surfaces & HYDRAULICS Total Hydrostatic Force on

Solution :
Solution
Considering 1 m length dF=pdA
F1 = 17, (9.81,,) (hX1)
Fr = 4.9051,2 kN dA=2x dy
r', = 9.81h(1.sX1)
tsy squared propelty of parabola:
Fz = 14.715ft kN v
*' =2' dy
lEMo = ol y3
F{h/3) + W(0.5) -Fz(1..5/2)= s
4.e05tP (tr/3) + s(0.6) - 1.4.71sh (0.2s; = 6 xz = tlt
1..6351f-11..04h+3=0
*=zJv /3
Solve ft by trial and error
h= 0.2748 m dF,=w122 (2r JM ) dvl
dF'= 2.311yf/,'2 dy
F 3

Problem 3 - 26 IdF =2,2.31


!.31yY y3 /2dy
irli
In Problem 3 - 25, find ft when the force against the "stop" is a maximum. J'
0 o
Iv'
I

s13
f== 2.317 lz ,rr2
Solution _t
l
v I
= 2.31(9.81) f 135/2 - Os/z1
[rMo = o] L5' I'
F lh / 3) + W (0.6) + P(1.s)- / 2) = o
f==147.3r ttl
P2(1..s
4.905tt2 (h/3) + s(0.6) + P(1.5)
t -'14.715h (0.751 = 0
Location:
P =1,.09h3 _7.359h+3
J

9=g.ZZ tP -7.358=0 F yo=


dh
lf = 2.25 9.81h 0
I,*oo
ft=1.5m 3

741.8 yo = 1 z dvl
tob.rr*'
Problem 3 - 27
3
Determine the force due waterto
acting on one side and its location on yo = 0.1,604
I the - parabolic gate shown using 0
F',,*
integration.
y,= 0.1.604lt,tn
I

"')'o
yr= 0.1.604 (2/4lg?/2'- Ouzl
yo= 2.14 mlelow the w.s.
)

li I -I ^ CHAPTER THREE FLUID MECHANICS C.XNPTT'R THREE


I
I Z+ Total Hydrostatic Force on Surfaces & HVDRAULTCS Total Hydrostatic Force on Surfaces 125
i

I Problem 3 - 28 Problem 3 - 29 (CE Board)


rl
l;l
il
In the figure shown, find the A dam is triangular in cross-section with the upstream face vertical. Water is
width D of the concrete dam flushed with the top. The dam is 8 m high and 6 m wide at the base and
necessary to prevent the dam weighs 2.4 tons per cubic meter. The coefficient of friction between the base
I

from sliding. The specific gravity and the foundation is 0.8. Determihe (a) the maximum and minimum unit
of concrete is 2.4 and the presoure on the foundation, and the (b) factors of safety against overturning
coefficient of friction between the and against sliding.
base of the dam and the
foundation is 0.4. Use 1.5 as the Solution
factor of safety against sliding. Is
the da{. also safe from Sp,g..ofconc,r.o..=
overturning ? Y
Sp.gr.of conc, ,,,n = ?!!J9 =2.4
Solution
. 1000
Consider 1 m length of dam
Consider L m length of dam
Wr=yrV,
W,= y(2.4)l(ax6)(1)I w=yrv
W,=74.4by =
Qx2.4) [r1011a11r1]

p=yhA W = 57.6 y where y = unit wt. of water

F =y(2.2\19.5)(1)I
F = 10.1?5y
P=yhA .trn w's'
t<(--- B = 6m
= 7(4)(8 x 1)
R' = f'= 10.125Y f =32y
,l Rv=W,
Rv= L4.4AV
&=P=32y
l
Ru=W=57.6,
rRY
rs..R,
= XM=W(4\
= 57.6y$)
0.4(14.4bt\ RM=?30.4y
1.5 =
1.0.1,25y
OM=P(8/3)
b=2.537 m = 32y(8/3)
" OM=85.33y
Fs.= RM
OM x=-RM-OM
Rv
B/5=lm U
Dc _ wc(b /2)
r(1.s)
7=
*-'a=- 85.33v
230.4v --'--' =2.519m<B/2
1,4.aQ.937)y(2.637 /2) 57.6y
,r"_ = 3.3 > 1 (Safe)
10.1257(1.5)
e=B/2- i
e=3-2.519 =0.481mc8/6
'-

I -' CHAPTERTHREE FLUID MECHANICS FLUID MECHANICS CHAPTER THREE


t27
I ZO Total Hydrostatic Force on Surfaces & HYDRAULICS Totat Hydrostatic Force on Surfaces
& HYDRAULICS

Figure:

'=-+(,"#)
-tl_ _s7.6(e.81) [, * e(0.+Sr)l
6 L'= 6 )
Using (+): q7 = - 139.47 kPa ) soil pressure at the toe
Using (-): 4p = - 48.88 kPa ) soil pressure at the heel

"" _ Rl _
FR, _o.s(S7.by)
,,n_
w
FS" = 1.44

RM 230.4y
FS, =
Ol/I 85.33y
FS' = 2.7

Problem 3 - 30 (CE Board May 1992)


Solution
A gravity dam of trapezoidal cross-section with one face vertical and
horizontal base is 22 m high and has a thickness of 4 m at the top. water A. NeglecHng hydrostatic uplift:
upstream stands 2 m below the crest of the dam. The specific gravity of I. Consider L m length of dam
masonry is 2.4 II. Forces
A. Neglecting hydrostatic uplift: Wr=fclrr= (Y x z.a)[(a)(zz)(r)]
'1. Find the base width B of the dam so that the resultant force will cut Wt=211.2y
the extreme edge of the middle third near the toe.
2. Compute the factors of safety against sliding and overturning. Wz= (r x2.4)11/z (B-4)(20X1)I
Use P = 6'5' Wz= 24By -96v
B. Considering uplift pressure to vary uniformly from full hydrostatic F=yhA=y(10)t(20)(1)I
pressure at the heel to zero at the toel F = 200t0
1. Find the base width B of the dam so that the resultant force will act at III. Reaction
the exkemity of the middle third near the toe.
2. compute the maximum and minimum compressive stresses acting &=rF,=P
against the base of the dam. & = 200ry

R, = XF, =Wr+ Wz
=217.2y + 24By -g6y
Rr=248Y +115.4
128 fl"1fi;il:*ff Force on surraces
FLUID
&
FLUID MECHANICS CHAPTER'THREE
Total Hydrostatic Force on Sirrfac€s 129
& I{YDRAULICS

|a'.1
^-u.atw
B =17.175m

Factors of Safety:
Factor of safety against sliding:
uR,,v
rc:'
Les
F R*

I
(0.s\[2a01,.175)y + "t"tl.zyj
I
20ll 200y
FS, = 0.9585

Factor of safety against overturning:


Uplift
pressure -oM
""=RM
diagram
i =a/3
- 16(11,175)2 y +_Y_.2(.9].175)y - 166.a,y

FSo= 2,97
IV. Moment.about the toe
RM = W1(B - 2) + Wzt$ 1a . +11 A. Considering hydrostatic uplifh
= 211.2y(B - z) + (zuB^t - s6y) 4)l
t+ @ _

= 211.28y - 422.4y t t6BLy - IZBBy + ]j;6y Ry=Wt+Wz-lf


RM = l6BLy + 83.28y - 1.66.4y = 24Bf + 115.1y _ tlBy
Ry= L4BY+1\5.2Y
OM= F(20/3)
=20uye0/s) W=W(B -z) +wzlt (B - 4)l
OM = 1233.32y .t
kM = 6B2y + B3.2By - 1.6G.4,1
V. Location of R: OM= F(20/3) + U(zB/s)
Ry7= RM-OM = 200y(20/3) + 10By (28/3)
OM= 5,678\ + 1333.33y
Since the resultant force will pass through the extreme edge of Rr 7 = RM-OM
the middle thirds near the toe, i = B/3. Then, (748y + 115.2^t) @ / 3) = 1682^t + 83.28y - 766.4y - (6.67 BLy + 1333. 33y)
4.6682 + M.8B -1,499.73 = 0
+ /g) = 16BLy + B:.2By _ 166.4y _ 1333.33y
11,5.2y)(B r-----
?aS,
SBzy + 39.48, = 168\ + B3.2By - 14gg.1gy , _ - 44.8 1 { (M.8\' - 4(4.56)(-1.499.73)
D-
882+U.9B-1499.23=0 z@s6\
B =13.766m
ql!',[''
l

i,

I
r 30 ;:HI;ffi:[:]: Force on surfaces FLUID MECHANICS
& HYDRAULICS
CHAPTERTHREE
Total Hydrostatic Force on Surfaces tt J
? tI

Foundation stress: Solution


ill, v =B/3
i = 13.766/3 = 4.59 m
F=Yu'iA
e=B/2- V =2.2943m = e.81(3)(6 x 1)
iii
rl F = 176.58 kN
il
,=-&l,r.!e)
, B\ B/ v= * $)=zm
;il' Wr = ycVt
Rv = :ri(ts.z o6xe.81) + 1 1s.2(e.81)
= 23.5[2(8X1)]
fl Ry = 3020.73 kN
ti
Wr = 376 kN
\l
Wz= T,Vz
,fl
o=_
'
3,020.72
l, * ez.zgEg)l = 73.slyz(2)(8\(1)j
.l 13.766 L 13.766 )
W: = 188 kN
Ir qr=-438.87kPa
qH=0kPa
xr=4-1/z(2)=3
t, xz= (2/3)(2) = 1.333 m

lr
Problem 3 - 31 (CE Board May 2002) &=F=176.58kN
I The section of A concrete gravity Ry = W, + Wz= 376 + 1.88
I dam shown in the figure. The RY = 564 kN
depth of water at the upstream side
is 6 m. Neglect hydrostatic uplift uR..Y
FS"='
-R,
and use unit weight of concrete
equal to 23.5 kN/fm3. Coefficient of 0.6,(56,4)
friction between the base of the FS, = =1.97.5
176.58
dam and the foundation is 0.5.
Determine the following: (a) factor RM=Wh+Wzxz
of safety against sliding, (b) the = 376(3) + 188(1.333)
factor of safety against RM= 1378.604 kN-m
overturning, and (c) the
OM= F xy
overturning moment acting against
= 176.ss(2)
the dam in kN-m.
OM = 353.16 kN-m ) overturning moment

"oM
""=RM
1378'6u
-
Pg^=
353.1,5
= 3.904
. -^ CHAPTERTHREE
CHAPTERTHREE
I 5Z ?3
Surfaces II r,-
Totat Hydrostatic Force on surfaces FLUID MECHANICS
& I{YDRAULICS Total Hydrostatic Force on
Problem 3:32 (CE Board November 2001)
Problem 3 - 33 (CE Board May 1986)
The section of a gravity dam is
The section of the masonry dam is as shown. The specific weight of water is
as shown in the figure. Assume
f 9,81 kN/mr and that of concrete is 23.54 kN/ma. Assuming uplift pressure
hydrostatic uplift to vary I
varies linearly from 7m El.52m
uniformly from full hydrostatic I maximum hydrostatic
uplift from the heel to zero at
pressure at the heel to zero at
the toe. Determine the total
I

the location of the drain,


I

reaction per unit length at the 60'


determine the (a) location of
base of the dam. Use sp. gr. of
the resultant force, (b) factor
concrete = 2.4.
of safety against sliding if
I coefficient of friction is 0.75,
Solution
J- ({ factor of safety against
lf= overturriing, (d) the shess at
'*R, the heel and at the toe, and (e)
the unit horizontal shearing
Consider l-foot length of dam:
shess at the base.
11,= F =yi A
= 52.a(30)(50 x 1) Solution
R, = 112,320 Ibs

Ry=W, +Wz+W-ll
Wt=yrVr
w1= (52.4 x 2.4\!+:(60X1)
Wt = 224,6401bs

Wz=f,Vz
wz= (62.4x2.4)9#$)(1)
w*z= 44,9281bs
rl !
!

W = T, ft = (62.4 x 2.$ YzQ$@0)(1) Wr


!
,!
W=7'1.,884.8lbs I
i
I
Wq = t* Vc = (62.4) Y2(30)(60)
Wa = 56,1.60Lbs
U = y,, V u = (62.4) i/z(60)(70)(1) = 131,040 lbs
R,, = 224,640 + M,gzg + 71,,994.g + 55150 - 131,040
Rr= 266,572.8lbs

R= (112,320\2 + (266,572.8)2 = 2892691bs per foot


riii,_

. -t;l CHAPTERTHREE
I 5+ Totat Hydrostatic Force on Surfaces
FLU'D MECHANICS
& ItfllBilf,:liANrcs rotarHvdros,,r,. rTf;J',tT,H#! I 35
('') t''o= Rtr/l= -TrTisB
I. Consider 1. m length of dam 683,900."12
oLt
Il. Forces FSo = 1'31
W't ='l' Vr
wt = 23.54lvz(s.2)(s2)(1)l = 3,183 kN (d) FoundationPressure:
wz= 23.54 Knp2)(1) = 8,569 kN e=B/2-i
e =38.2/2-13.2=5.9 m< B/6
Wt= 23.541y,(26)(s2)(1)l = 15,913 kN
W+ = 9.81[1/z(5X50)(1)] = 1,226.3 kN R,, ( 6e)
a=-
. B\'| 1+-lB)
U =1/z(4e0.5)(%.2)(1) = 5,6e0 kN -l
23,201..3[, * ots.sl
F= yiA = 9.81(25)[s0(1)] = L2,263 kN -'
o =_
38.2 L 38.2 I

III. Reaction
Stress at the toe, (use "+'r);
R,=F=12263kN
q= -1,170'27kPa
Rv=Wr+Wz+W+Wt-U
= 3,183 + 8,569 + 15,913 + 1,,226.3 - 5,690 Stress at the heel, (use "J'1
Ry = 23,201.3 kN qr, = -44.52kPa

IV. Moment (e) Unit horizontal shearing stress, S,


RM = Wr (34.73) ++ Wt(tZ .ZZ) + Ws(36.53)
vt1r129.5)
s.= R,
= 3,1,83(34.73) + 8,569(29.5) + 15,913(17.33) + 1,226.3(36.53) ' Abrr,-72'263
38'2(1)
=321kpa
RM = 583,900.12 kN-m
oM=F(50/3)+U(30.44
12,263 (50 / 3) + 5,690 (30.44 Problem 3'34
OM= 377,758 kN-m 'I'he submerged curve AB is one
quarter of a circle of radius 2 m
V. Location of R,
and is located on the lower
R, 7 = kM-OM
corner of a tank as shown. The
?3,201.3 7 = 683,900.12-377,758
Iength of the tank perpendicular
i =13.2m to the sketch is 4 m. Find the
magnitude and location of the
(a) The resultant force is 13.2 m frort'r the toe.
horizontal and vertical
components of the total force
uR..
'ii
(b) Fs,= +K, acting onAB.

0.75(23,201.3)
FS, = = 7.42
12,263
,.-nl"---- l

iri
:lr
I -'
I 5()
CHAPTERTHREE
Total Hydrostatic Force on Surfaces
FLUID MECHANICS
& HYDRAULICS
FLUID MECHANICS CHAPr_ER
Total Hydrostatic Force on
rHREE
Surfaces It J t
& HYDRAULICS
"7
;ili
,,I
Solution
Fr=yi A
Another way of solving r :

Since unit pressure is always normal to the


il ru = e.81(s)[(aX2)]
surface and a normal to the circle passes
-T-
Fs = 392.4 kN through its center, then the total force F
ti y=1,+e shall also pass through the center of the I

4m
t"6 circle O, hence the moment about O due to
li V=5m F or due to Fs and Fv is zero.
Ay

I - _ +1213 1tz lIMo = 61


, t(4)(2)1(5) -T- Fvi-FpY=g
ir e = 0.067 m 437 :t3 i = 392.4(1..067)
t = Q.9578 m
li'i Y=1+0.067
Y = "1.057 rn Note: This is true to all cylindrical or spherical surfaces.
lr
Therefore; F11 is acting 1.067 m below B Problem 3 - 35 (€E Board)
I The crest . gate shown
Fv = Weightaaco
I Fv = yVaaco
consists of a cylindrical
I
Vaeco = 4(A)
surface of which AB is the
A=Ar+n, base supported by a

At=@)(2)=8m2 structural frame hinged at


A2 = rln n(2)2 = 3.1.4 m2
O. The length of the gate
is 10 m. Compute the
A=8+3.14 magnitude and location of
A= 11".14 m2
.the horizontal and vertical
components of the total
Vaaco = 4(11.14) = 44.56 rn3
pressure on AB.
Fv = 9.81.(44.56)
Fv = 43213 kN 10cos60" = 5m
Solution
Location if Fv:
Ai = At x.r * Azxz
rr=1m
I^=
41 =
4(2\
_:__1
E
\r,
li -3n 3n o
ad
ii lz = 0.849 m il
o
rO
7'1.1.4 V = 8(1) + 3.14(0.&19) c
'6
x = 0.957 m o

Therefore; Fy is acting 0.957 to the right of A


L=10m
.FIUID THREE
t 38 ;L:i;;l;H:[ Force on Surraces
MECHANICS
& HYDRAULICS
FLUID MECHANICS
& HYDRAULICS
CHAPTER
Total Hydrostatic Force on Surfaces l4,q
tr '

FH=yh A 3olution -
Fu = 9.81. (4.33) [10(8.56)] Considering 1 meter length:
Fr-r = 3679 kN Fp=yh A
y= \(s.6e=2.887rn FH=9.81(3)(6x1)
Fs = 176.58 KN
Therefore; Fs is acting 2.88V mabove O
Fv=tV,
A,=Ar*tor-Atria.gte
Fv=yV$c
Vasc=Veoac-VaoB
,r(6)2 (60')
A"" = - v26\2sin 6oo
360'
vasc= $
1s.ooy. x10 *vz(10)z [00.5.] " ro A"=3.26m2
Vaac= 125.9 rnl
Fv=9.81(3.26x1)
Fv = 9,87(125.90) Fv = 31.98 KN
kN
Fv = 1235
F= W *P;
Moment about O due to Fn and Fy = Q F= 1@ssf *(31-es)'
Fv (x) = Fu tg) F = 179.45 KN
1235 x=3679(2.BZn
x=8.57m
Therefore; Fy is acting 8.57 m to from O Problem 3 - 37
Calculate the magnitude of the
resultant pressure on a l-ft-wide strip
Problem 3 - 36 (CE May 1999) of a semicircular taintor gate shown
Calculate the magnitude in Figure-12.
of the
resultant force per meter length due
to water acting on the radial tainter
gate shown in Figure 021.

Solution
Fu = p,cA
i
P, = (62.4x 2.s)(5 x 1) = /36 16t

,I Fv=lVzulc
Fv = 62.4 x If (s)'z(l)] = 1225tbs

F= .ffif *(Frf
P = ,,1 1ZSO12 + (1225\2 = 1452lbs
r"

li II 4n CHAPTER THREE CHA.Pr_ETTIITE_!


Surfaces Il4l
Iv FLUID MECHANICI FIUID MECHANICS
Total Hydrostatic Force on Surfaces T I
ll
lli
& HYDRAULICT I HYDRAUTICS Total Hydrostatic Force on

lr Problem 3 - 38 39
Froblem 3 - (CE Board November 1993)
I
Determine the magnitude of
Itr tlrc figure shown, the 1.20 m
I

the horizontal and vertical rliarttctcr cylinder, 1.20 m long is


components of the total force allt'tl upon by water on the left and
per meter length acting on the rrll lrlving sp. gr. of 0.80 on the right.
three-quarter cylinder gate l)1.lt,rrnine the components of the
shown.
r'+.rr.tion at B if the cylinder weighs
l(,.(r2 kN.

,r,=rh A
Fm = 9.8\(1.2) (1.2 x 1'.2)
Far = 16.95 kN

Fw = YVt
F w = 9.81lVz a (0.6),(1.2)l

Fvr = 6.657 kN
Fuz=Th A= (g.51, x 0.8)(0.6)(1.2 x 1.2)
Fsz = 6.78 kN

Fvz=^tVz= (9.81 x 0.8)[% n (0.6)'?(1.2)]


Fvz = 5.32 kN

Fn=yh A lIf, = 91
ra = e.81(3)[(1)(2)] Fm-Fuz-RaH=0
Fs = 58.86 kN Rsn=16.95-6.78
RsH = 10.17 kN

Rav+F1n+Fvz-W=0
Rsv=19.62-6.657 -5.32
Rav = 7.64 kN

Fv =tVol
Fv = 9.81[4(2X1) + 0.7sln(2)r(1)]
Fv = L70.94 kN
CHAPTER TIITEE 43
Surfaces II -Ti
.I CHAPTERTHREE FLUID MECHANICS FLUTD MECHANICS
I +Z
^- Total Hydrostatic Force on Surfaces & HYDRAULICS & HYDRAULICS Total Hydrostatic Force on

Problem 3 - 40 F2 = (9.81 x 0.82)(0.00628)


Fz = 0.0505 kN
An inverted conical plug 400 mm diameter and 300 mm long closes a 200 mm
diameter circular hole at the bottom of a tank containing 600 mm of oil having Fo=Ft-Fz
sp. gr. of 0.82. Determine the total vertical force acting on the plug. Fo=0.1L4 - 0.0505
F, = 0.0635 kN = 53.5 N downward
Solution

:"",l Problem 3 - 41
A 2 m diameter horizontal cylinder 2 m long plugs a 1m by 2m rectangular_
hole at the bottom of a tank. With what force is the cylind,er pressed against
the bottom of the tank due to the 4-m depth of water?

Solution

h = 2* (L cos 30")
lu=L.732m
hz=4-ht
hz= 2.258 m
I

li, Ft = lV.r
Vt=Atx2
1, Area, Ar = Area bf rectangle DEFG - A4
l

j
I
Area or sesment, A. = '(?:f:0') - v,(l)(1.)sin 60o
I

il,
ll L, n= TVt Area of 6egment, Ar = 0.09059 m2
'il,l Fl = (e.81 x 0.82)[r(0.1),(0.as)]
1r[rl] Fr= 0.114 kN Area,Ar =2.1774m2
1,' V = 2.1774(2) = 4.355 me
ti l
Fz= YVt
Fr = 9.81(4.3s5)
Vz= VFrustum - Vcyltnd",
I

I
h=42.72kN
%= *(0.2)(0.1)+(0,1)rl - r(0.1),(0.1s)
I ry[0.21, Fz= Fs= yVz
= 0.00528 ma Vz= Azx 2
ir Vz
i

h
CI'IAPTERTHREE t rF
t 44 fI*iI;H:[:J: Force on surraces FLUID MECHANICS
& HYDRAULICS
FLUID MECHANICS
& HYDRAULICS surfaces I '+)
Total Hydrostatic Force on

I
r(1)-?0') fs=yh A
Area of segment,A, = -%G)$)sin 120o
Fu = e .81(6.12)t$.24) (11
Area of segment, Az= 0.614m2 Fs = 254.56 kN
vz= 0.61,4(2) = 1,.2?3 mg
Fz=Fr=9.81,(1,.228) Fv = Y Vrnua"a
Fz = Fr = 12.05 kN %fr"a"a = (Asemictrcle + {610"ro1,1) x 1

%noa"a = [*n(e)' +ff{+.2+))0)


Net force = p, - F2- Ft = 40.1 m3
%nua"a
Net force = 42.72 - 12.05 = 12.05
Net force = 18.52 kN Fv = 9.81(40.1)
Fv = 393.38 kN

Problem 3 - 42
Problem 3 - 43
In the figure shown,
determine,the horizontal and The gate shown is a
vertical components of the quarter circle 2.5 m
total force acting on the wide. Find the force F
cylinder per m of its length. iust sufficient to prevent
rotation about hinge B.
Neglect the weight of
the gate.

Solution
6 cos45o
= 4.24m

Solution

Fs=yh A
:{
t;$ Fr = 9.81(1)(2.5 x,2)
oll
Fs = 49.05 kN

Fv = Y Vasc
F v = 9.81[(2 x 2) - 0.25n(2)z
Fv ='21.05 kN
I ,i ()
, CHAPTER THREE FLUID MECHANICS FLUID MECHANICS CHAPTER THREE I a7
I 'f rotal Hydrostatic Force on surfaces & HYDRAULICS, & HYDRAULTCS Total Hydrostatic Force on Surfaces ' t '

Solve for z and x: Forces due to oil:


Since the surface is circular, LMs = 0 due to Fa and Fv Fuo = Prgo A
Fv(z) = Fu(2/3\ Fse = (9.81 x 0.80)(7 -'1.273) x 1/zn(3)2
2L.05(z) = 4e.0s(2/3) Fso = 635.4 kN
z=1.55m
x=2-z=2-1..55 Fvo = lu Vn
.r = 0.45 m V, = Volume of imaginary oil above the surface
[IMs = 0] V, = Volume of half cylinder - Volume of % sphere
FH (2/3) + Fv (r) - F(2) = g -
V, = Yzn(3)2 (7) re f n(3)3
2F = 4e.05(2/ 3) + 21.0s(0.4s) V,=70.686rr.t
F = 21.09 kN
Fys = (9.81x 0.80)(70.686,)
Fvo = 554.74 kN

Problem 3 - 44 Forces due to,water:


he cylindrical tank shown has a Fuvv= Pre* A
hemispherical end cap. pr* = (9.81 x 0.8)(4 + 9.81('1,.273)l x Ym(3)2
I Compute the horizontal and Fr+w = 953.19 kN
ll vertiial components of the total
I
force due to oil and water acting Fvw = Weight of real and imaginary oil above the surface
lrl
on the hemisphere. + weight of real water above the surface
t.'i FyyT = (9.81 x 0.8)x lzn(3)2(7) + 9.81 x
thf r(3)g
Fvw = 1,054.01
il,l
1

Total horizontal force, FH = Fyo + lLilN


I

Total horizontal force,'Fn = 635.4 + 953.19


lll Total horizontal force, Fri = 1,588.59 kN )
Solution
Total vertical force, Fv = Fvw - Fvo
Total vertical force, Fv = 1,054.01 - 554.74
Total vertical force, Fv = 499.27 kN

Another way to solve fof the total vertical forcq, Frz

Fv = weight of fluid within the hemisphere


F, = yo Vn * yro V,u
Fy = (9.81x0.4)tlx in (3)')l + e.s1[]x i" (3)')l
Fv = 499.27 kN

rl
il
.
-
I
l

t 48 ;IfiI;l':T[:[ Force on surraces


FLUID MECHANICS
&
FLUID MECHAN'CS
& HYDRAULICS
CHAPTER THREE
Total Hydrostatic Force on Surfaces 149
i, Problem 3 - 45
Problem 3 - 46
Pressurized water fills the tank shown in the figure. Compute
the Determine the force required to open the quarter-cylinder gate shown. The
hydrostatic force acting on the hemispherical surface.
weight of the gate is 50 kN acting 1.2 m to the right of O.

Hemispherical
surface

Solution
Convert 100 kPa to its
equivalent pressure
head, }* lh Solution
,- -P_
,req _

.
v
100
IL Since the gate has circular surface,
the total water pressure passes
through point O which is also the
ftm-
' 9.81 location of the hinge, therefore the
-
h"c= 10.194 rn T,I, moment due to water pressure
about the hinge is zero.
h=10.194-5
h=5.194m lxMo = 0l r=2,5m
l_i_
^
100 kPa
F(2.5) = s0(1.2) + Fr(0)
F=24kN
F = Weight of imaginary water above the hemispherical
surface
F=Y'v,' Problem 3 - 47
= Volume of cylinder + Volume of hemisphere
Vu,
A hemispherical dome shown is filled with oil (s = 0.9) and is attached to the
V*= n(2)2(5.1ea) +
!x floor by eight diametrically opposed bolts. What force in each bolt is required
tr(2)3
V,,= $).Q/g sP to hold the dome down, if the dome weighs 50 kN?
F = 9.81(82.025)
F = 8M.7 kN

L
r -! _l

CHAPTER THREE FLUID MECHANICS CHAPTERTHREE I F I


r50 Total Hydrostatic Force on Surfaces & HYDRAULICS surfaces I ) I
Total Hydrostatic Force on

Solution
[IFy=0]
F+For-Frir=0
F=F"ir-F"it
Fnil = YVoil the cone
"uote
P,u = (9.81 x0.8) [r(0.805)r(5)
- { r(0.80s)'?(3)l
Foir = 63'9L kN
Fat = Pair A
F"i' = 20 tt $.51)'z I = 40.72 kN
F=63.91,-40.72
Solution F = 23.19 kN
FY = Y %.uginary oil above the dome

P, = (e.81x0.e)[n(2), (8) -lrQ1t1


Problem 3 - 49
Fv = 739.66 kN
A 300 mm diameter steel pipe L2 mm thick carries water under a head of 50 m
8F5o11*W=Fv of water. Detetmine the stress in the steel.
. _ ___
.*,,_ 739.66-50
g Solution
Fta. = 85.2 kN p,=
*l
9'81(sox3oo)
Problem 3 - 48
s, - 2(1'2)
= 6131.25 kPa

Determine the force F Sr = 5.13 MPa


required to hold tJre cone
shown. Neglect the weight of
the cone. Problem 3 - 50
Determine the required thickness of a 450 mm diameter steel pipe to carry a
m?omum pressure of 5500 kPa if the allowable working stress of steel h 124 MPa.

Solution ;

$,= tl p$so)
. 124x looo -
2t
t =9.98 mm say 10 mm

t
q

f F-i CHAPTER THREE FtUId MECHANICS CHAPTERTHREE . ?-


I )Z Total Hydrostatic Force on Surfaces & HYDRAULTCS rotat Hydrostatic eTl"i;t##: 153
Problem 3 - 51 Pipe diameter, D = 6 m = 5000 r-nm
Determine the stress at the walls of a 200 mm diameter pipe, L0 mm Maximum pressure the tank (at bottom), p = f oirh
under a pressure of 150 m of water and submerged to a depth of 20 m in
water.
2(L10x103X300) ',

Solution "_
-----------------
s4.e36(6000)
S = 200.?3run say 200 mm

p=pinside-poutside
p = e.81(150) - e.81(1.03)(20)
p = 1269.4 kPa = 1.269 MPa
A thin-walled hallow sphere 3.5 m in didmeter holds helium gas at L700 kPa.
- = -----J------3
5'1
7.2o9Q00\
=12.69MPa Determine the rninimum wall thickness of the sphere if its allowable slress is
2(10) 60 MPa.

Solution :

pD
Wall'skess, S,=
A 100-mm-ID steel pipe has a 5 mm wall thickness. For an allowable tensile T
stress of 80 MPa, what maximum pressure can the pipe withstand? 1,700(3.5 x L000)
60,000 =
Solution
t=24.79mm
rt,=
*l
Problem 3 - 55
3s = P(1oo)
2(6) A vertical cylindrical tank is 2 meters in diameter and 3 meters high. Its sides
p =9.6 MPa = 9,500 kPa are held in position by means of two steel hoops, one at the top and the other
at the bottom. If the tank is filled with water to a depth of 2.1. m, determine
the tensile stress in each hoop.
Problem 3 - 53
A wooden storage vat is 6 m in diameter and is filled with T m of oil, s = 0.g. Solution
The wood staves are bound by flat steel bands, 50 mm wide by 5 mm thick,
whose allowable tensile stress is 110 MPa. what is_the required spacing of the

Solution

Spacing of hoops, g = ZSA|


.pio,, l

Allowable tensile stress of hoops, Sr = 110 Mpa


Cross-sectional area of hoops, Ar = 50(6) = 300 mm2
7--

154 ;L:|I;HT[:[ Force on surfaces


FLUID MECHANICS
& HYDRAULICS
CHAPTER THREE
Total Hydrostatic Force on Surfaces r55
[I M", = 61 Problem 3 - 57
2Tz(3) = F(2.3)
A cylindrical container 8 m high and 3 m in diameter is reinforced with two
Tz = 0.3833F ) Eq. (1)
lroops L meter from each end. When it is filled with water, what is the tension
f=yna in each hoop due to water?
F= e.sl(+
)l(2x2.1)l = 43.26 kN Sotution
In Eq. (1)
Tz= 0.3833(43.26)

'l
Tz = 16.58 kN (tension in the bottom hoop)
-T----T-
I tm
lI FH:01
272+ 27r= P
l-1-
+r'
16l3
2Tr=P-rr, I l3/3

2T1= 43.25 - 2(16.58)


Tr = 5.05 kN (tension in the top hoop)
lt
8m

I
6-m

t-'
lll
I
'i-:
sl3

Problem 3 - 56
A vertical cylindrical iank, open at the top, is filled with a liquid. Its sides are
held in position by means of two steel hoops, one at the top and the other at p=yhA
the bottom. Determine the ratio of the stress in the upper hoop to that in the F = e.81(8/2)[8(3)]
lower hoop. F = 941.76 kN

[ZMopr,oop = 0]
Solution
272(6) = F (13;/3)
Tz= 13F/36
Ratio=Tt/Tz Tz= 13(941.76)/36
Tz = 340.08 kN
[EMt"p = 0]
2r2(h) = F(2h/3) [XMuotto^ r,oop = 0]
Tz= F/3 2r{6) = F(5/3)
Tr = 5F /36 = 5(941.75)/36
Tr = L30.8 kN
[XMuotto^ = 0]
Zr{h) = F(h/Z)
Tt= F/6
Problem 3 - 58 (CE Board November f982)
F/6 A cylindrical tank with its axis vertical is 1 meter in diameter and 5 m high. It
Ratio= =o.s
F/3 is held together by two steel hoops, one at the top and the other at the bottom.
ThreeliquidsA,B,andChavingsp.gr.ofl.0,2.0,and3.0,respecfivelyfillthis
tank, each having a depth ot'1..20 m. On the surface of A there is ahnospheric
pressure. Find the tensile stress in each hoop if each has a cross-sectional area
of 1250 mm2.
156 ;#i;;5,:H:,: Force on surraces CHAPTER THREE
Surfaces I' rtr^Z
FLUID MECHANICS FLUID MECHANICS
& HYDRAULICS & HYDRAULICS Total Hydrostatic Force on '
Solution

r
Tz = 3.6(9810) = 35316 N
m
2Tr pr=0 T. 35316 -
A2 7250
Liquid A
s=1,0
Skess, Sz = 28.25 MPa ) stress in bottom hoop

=- [IFu = 0]
3.6 m Liquid B 271 + 77r= Fr + Fz + F3 + F4 + F5
s=2.0 1.2 m 2T1= 9.7r, + 1.44y + 1.44y + 4.32y + 2.L6y - 2(3.6y)
Tt = 1.44y
Tt = 1,.44(9810)

L, Tt = 1.4L26.4N

sh'ess.s,= I
A1-14'126'4
1.,250

fi=o Stress, Sr =L1.3 MPa ) skess in top hoop

Pz= pr + Yaha
pz=0+ (yxt)(t.2)=1.2y
Problem 3 - 59
Pt=P2+Ysl1o An open cylindrical tank of 1..86 m2 cross:sectional area and 3.05 m high
h = 1.2y + (yx2)(1.2) = 3.6y water. Into it is lowered another smaller tank of the
contains 2,ti31 liters of
same height but of 093 rfr cross-section in the inverted position, allowing its
P4=h+ychc
pt = 3.6y + (yx3)(1.2) = 7 )y open end to rest on the bottom of the bigger tank. Determine the maximum
lcnsion per vertical millimeters on the sides of the bigger tank, Neglect the
h =r/z@z\(1.2[1)
thickness of the metal forming the inner,tank and assume normal barometric
h = th(1.2:i (1..2) (1\ = 0.72y
pressure.
Fz = pzl1..2)(1)
Fz= 1.2y(7.2)(1) = 1.aay Solution
h= Vz(ps - pz)(1.2)(1) 1.86 m2

h = l/z?.6y 1't.2y)(1.2)(1) ='t.44y 0.93 m2

Fq = p{1.2)(1)
ir Ft = 3.6y (1.2) (L) = 4.22y
,l
Fs=lh(pt - p)(1.2)(t)
Itiliii; Ft = lz(7 .2y - z.6y)(1..2)(1) = Z:!.6t0
i

[EM"p = 0]
1.6QD = Fr(0.s) + Fz(1.8) + Fs(2) + Fr(3) + Fs(3.2)
7.2T2= 0.22y(0.8) + 1,.My(7.8) + Laaye) + 4.32y(i) + 2.16y(3.2)
Tz= 3.6y O Before lowering o After lbwering
l

.iiu
I 58 ;IfiI;I.:H:[ Force on surraces
FLUID MECHANICS
& HYDRAULICS
CHAPTER THREE
Total Hydrostatic Force on Surfaces t59
In Figure G) Problem 3 - 60 (CE Board November L977)
Volume of water = (1.86 - 0.93)(b + h) + 0.93b = # An iceberg having specific gravity of 0.92 is floating on salt water of sp. gr.
"t.86b + 0.93h -- 2.831 1.03. If the volume of ice above the water surface is 1000 cu. m., what is the
2h+lt=3.044 total volume of the ice?
b = 1.522 - 0.5h ) Eq. (1)
Solution
lpt Vt = pz Vz) Let V = total volume of ice
pt = 101.325 kPa (atmospheric pressure) Vb = volume displaced
Yl = 0.93(3.05) = 2.8365 m3 Vo= V -'l'000
pz= 101.325 +.9.81.h
v2= 0.93(3.05 - b) LV.e = Ti.e V = (9.87x0.92)(1)
Wi,"= 9.0252 V
7o1.s2s(2.836s) = (1 01.32s + 9.81/r) [0.93(3.0s - b)] BF = |r""*."t", fo
309.04 = 309.04 -'10't.325b + 29.921t - 9.81.bh BF (9.81x10.3XY' 1000)
29.92h - 1.01..325b - 9.81.bh = 0 =
BF=10.1043(v-1000)
29.92h - 1 01.325(1.522 - 0.51 1 - 9.8L(L.s22 - 0.5/r)/, = 0 [EFv = 0]
29.92h - 754.22 + 50.66h = 74.93h + 4.905ll = 0 Wx.= BF
4.905h2 + 65.65h - 154.22 = 0 9.02s2V = 10.1043(V - 1000)
1,.0797 = 10104.3
h= -65.65!@ = 2.039 V =9,364cu,m.
2(4.e}s)
b = 0.5027 m Another Solution:
H=b+h=2.542m For homogeneous solid body floating on a homogeneous liquid:
sbodY YbodY
The maximum tensile stress occurs at the bottom of the tank. vo = vuoay = yuoay

1
sliquirt Y Iquid

p=yH=9.81(2.542)
p = 24.937 kPa = 0.024937 MPa
then; V-1000 =ffiv
T 0.106796V = 1000
Tension, T: V =9,364ca.m.
2T = pD(l)
f Dz = 1.86 m2
D=1,539m=1,539mnl Problem 3 - 61 (CE Board May 2003, Nov 2002, May 200O, Nov 1992)
A block of wood 0.60 mx m x /r meters in dimension was thrown into the
0.60
2T = 0.024937 (1,,539) (1) water and floats with 0.18 m projecting above the water surface. The same block
T=19.2N was thrown into a container of a liquid having a specific gravity of 0.90 and it
floats with 0.L4 m projecting above the surface. .Determine the following:
(a) the value of /r,
(b) the specific gravity of the block, and
(c) the weight of the block.
l60 ;I*i;;ilH:[ Force on Surraces
FLUID MECHANICS
& HYDRAULICS
FLUID MECHANICS
& HYDRAULICS
CHAPTER THREE
Total Hydrostatic Force on Surfaces l6l
Solution Wrron"
In Water: Istone - -:-
Vstone

Draft =
Jwood
7'
. 460
S*,ater tstone = 28,204 N/m3
0.0153

c
rwood
h_ 0.lg _ 7,
1 Problem 3 - 63 (CE Board May 1993)
S*noa /z = ft - 0.18 ) Eq. (1)
A body having a sp. gr. of 0.7 floats on a liquid of sp. gr. 0.8. The volume of
the body above the liquid surface is what percent of its total volume?
ln another liquid
S*oo'1 Solution
Draft = 7,
Stiqui.l
Vo= fuVo"ay
Si,ooa sliquid
tt_0.14- 7,
0.9
S*,o4 /r = 0.9h - 0.126 ) Eq. (2)
Vr= #Vuo,ry = 0.875V6"6y
In other liquid (S = 0.9)
Since the volume of the body displaced (below the liquid surface) is 0.875
[S*ooa /t = S*oo,1 /t]
/,-0.18=0.9h-0.126 or 87.5% of its total volume, then the volume of the body above the liquid
h = 0.54 m ) height of the block surface is 12.5o/o of its total volume.

Substitute /r to Eq. (1):


S*",a(0.54) = 0.54 - 0.18 Problem 3 - 64 (CE November L997)
S*uoa = 0.557 ) Specific gravity of wood
A block of wood 0.20 m thick is floating in sea water. The specific gravity of
Weight of block = |wood Vbrock wood is 0.55 while that of seawater is 1.03. Find the minimum area of a block
Weightof block = (9.81 x0.667)l(0.6 x 0.6)(0.s4)l which will support a man weighing 80 kg.
Weight of block = 1.272 kN
Sotution
Wr..rAN = 80 kg
Problerm 3 - 62 ,I
A stone weighs 460 N in air. When subrnerged in water, it weighs 300 N. 'Wwmo
Find the volume and specific gravity of the stone. I ,1, o.z m

Solution
Weight of stone = 460 N
Weight of stone in water = 300 N [XFv = 0]
BF=W^"n*l/Vwood
Buoyant force, BF = 460 - 300 = 160 N
= LV^"n f y*ood Vwood
yrn, V*ood

[BF = y*ur". %ron"] (1000 x 1..03) V**a = 80 + (1000 x 0.65) V*ooa


150 = 9810 (%t"*) Y*ood = 0.2105 m3 = Area x 0.2
V.ton" = 0.0L63 cu. m. Area = 1..05 square meter
I
rT-
i
I

CHAPTER THREE
162 Total Hydrostatic Force on Surfaces FLUID MECHANICS FLUID MECHANICS
CHAPTER THREE
Problem 3 - 65 (CE Novemb
& HYDRAULICS & HYDRAULICS Total Hydrostatic Force on Surfaces
- t63
er 1997)
A cubc of wood (s'g' = 0.60) has g-in Problem 3 - 67
sides.
of the force required to hold the Cornpute the r.*agnitude and A uniforrn block of steel (s = 7.g5) .I7t **-
l,;;::il"^ wood comptetety su-bmerged in ---
will float at a mercury-water Water
interface as shown in Figure 27.
Solution What is the ratio of the distar.rces
1,1
t "a" and"b" for this condition?
Weight of wood = (62.4x 0.60) (+I
= 15.295 lbs
,i Buoyant force when
submerged in water:
I
BF : 62.4(+I = r::j;l[:t
Required force = Z3.g2S _ 1l.7gs
Required force = 10.53 lbs downward Figure 27
Solution

Probtem 3 - 66 GE G;2oo-0,
I
The block shown in Figure 04 t2' x 12'

I weighs 35,000 lbs. Find the


value of /r.
I LetA be the horizontal cross-sectional
I area of the block.

I
B\+ Sf2=1,1
fu,V\r,,*fnVor,=ysV
I
9.81(Ax a) + (9.81 x 13.56)(4 x D) = (9.81 x 7.8s)[A(a + b)l
Figure 04 a + 13.56b =7.85a + 7.85b
Solution
5.71b = 6.85 a
From the figure shown:
a/b = 0.834
llr, = o,
B\ + BPr= 35,000
Probtem3-68(eEMm98,
BFt = Tol Vo
B\ = (62.4x0.8X12x12x3) If a 5-kg steel prate is attached to one end of a
0.1 m x 0.3 m x 1.20 m wooden
Bh = 21,,565.44ibs il::f 3,ote,
wfat is the- rength of the pole above water? use s.g.
Neglect buoyant force on steel
or *ooa of 0.50.
21,565.44 + BFz= 35,000
BFz= 73,434.56hbs

BFz = T,,, Vo
13,424.56 = 62.a
[e2)(12) ttl
h ='1.495 ft
164 fL:iJ;H?.:ff Force on surraces
FLUID MECHANICS FLUID MECHANICS CHAPTERTHREE . .
Surfaces I (l5-
& HYDRAULICS & HYDRAULICS Total Hydrostatic Force on
So.lution
' Neglecting the buoyant force on steel: Problem 3 - 70
BF*noa = TVsteel i LV*"r,.l A wooden buoy (s.g. = 0.62) is 50 mm by 50 mm by 3 m long
is made to float
1000(0.1 x0.3xJ/)=5+ in sea water (s.g. = 1.025). How many N of steel (s.g. 7.85)
= should be
x 0.3 x 1.2] attached to the bottom to make the buoy float with
^1000(0.5)[0.i exactly 450 mm exposed
y = 0.77 nr above the water surface?

h ='1.2
- !/ Solution
It=1.2-0.77
It = 0.43 m IFy=g
BF"t""r + BF*ooa -W*ood- I/1y's6""i =g
BF"t*t = Y.- %t*r
BF,t".r = (9810 x 1.025)%t*r
BFrt*r = 10,055.25 %t"o N
BF*ood = l"n Vo
Problem 3 - 69 BF*ooa= (981.0 x 1.025)[(0.05)r(2.55)]
If a S-kg steel plate is attached to one end BF*ooa = 54.1 N
of a 0.1 mx0.3mx1.20mwooden
pole, what is the ler.rgth of the pole above W*ood= |wood Vwood
water? Use sp. gr. of wood of 0.50
arld that of steel 7.g5. LVwood = (9810 x 0.6,2)[0.05)r(3)]
LVwood = 45.62 N
Solution
LVrt*r = hrur %t".r
W*ood = (1000 x 0.5X0.1 x 0.3 x 1.2)
W*ooa = 18 Kg = (9810 x 7.85) V,ua
LV.t""r
W,t*r = 77008.5 %t.l
W'r"a = 5 kg
10055.25 %tu"r + 64.1" - 45.62 - Z70Oq5
%t""r = 0
BFw=1000(0.1x0.2xd) 65953.25 %eu = 18.48
BFyy= 39 4 %t".t = 0.000276m3
BFs = 19gO ,. Id.t*r = 9810 (7. 85X0. 0027 G)
I//s = (1000 x 7.85) V5 = g Wrt""t = 21.255 N
Vs = 0.000637 ms
BFs= 0.637 Kg

W*ord* Wrt".t= BFg+gp* Problem 3 - 71


18+5=0.637+ZOA of lead (sp. g.' 11.3) is tied to a 130 cc of cork whose specific
iritrlr
d = 0.745m l:'"::
0'25' They float just submerged in water. what is the weight
gravity is
of the lead?
x=1..2-d
r = 0.455 m
CHAPTER THREE FLUID MECHANICS FLUID MECHANICS CHAPTER THREE
167
l

I
I
I OO Total Hydrostatic Force on Surfaces & HYDRAULICS & HYDRAULICS Total Hydrostatic Force on Surfaces
i

,l I Solution Solution
ri
I
lIFv = oI
Wc+Wr=BFc+BFr
Wc= Yc Vc
Ws = (1x 0.25)(130)
Wc= 32.5 grarns
i
BFc = yu, Vc
BFc = (1Xi30)
BF6 = 130 grm

Wr=tuVt
Wt= (1x 11.3) Vi
1,, , Wr=11.3 Vt
I BFr - \u' Vt
BFr = (1) Vt= yt
32.5 + 11.3 VL= 130 + VL
Vr = 9.47 cc
(a) Lead is fastened outside the cllinder (b) Lead is placed inside the cylinder
Wy = 1.1,.3(9.47)
Wt= 706.97 grams
(a)Lead is fastened outside
BFc= y* Vo
Problem 3 - 72 (CE November 1993) BFc = 9'311* (1)'z(1's)l

A hallow cylinder 1 m in diameter and 2 m high weighs 3825 N. (a) How BFc = 11'55 U*
many kN of lead weighing 110 kN/m3 must be fastened to the outside bottom BFt= y* Vr
of the cylinder to make it float with 1.5 m submerged in water? (b) How many BFY= 9'g1Y'
kN of lead if it is placed inside the cylinder?
Wr= ytVt = L10Vr-

[XFv=0]
BF6 + fif't =Wc+WL
11.55 + 9.B7Vt = Z.B2S + 1t OVr
Vr= 0.0772n:F
Wr = 110(0.07727 = g-4g U*

(b) Lead is inside the cylinder


[XFv = 0l
Wt+ Wc= BFc
Wr+ 3.825 = 1L.56
Wt=7.735kN
t 68 ;Ifl;;:,:H:[ Force on surraces
FTUID MECHANICS
& HYDRAULICS
FLUID MECHANICS
& HYDRAULICS
CHAPTER THREE
Total Hydrostatic Force on'Surfaces 169
Problem 3 - 73 Solution
A stone cube 280 run on each side and weighing 425 N is lowered into a tank
containing a layer of water 1.50 m thick over a layer of mercury. Determine
s.g-=.0'8
the position of the block when it has reached equilibrium.

Solution
W= 425 N
BFp = yyV6p1
BFpl= (9,810 x 13.6)[0.28,(x)]
BFu ='10,459.81 x

BFw=yrvVorv
BFr,r= 9,616t(0.28)'?(0.28 - r)l
BFw=769.L(0.28-x)

3p = (62.4x7,a)l(2.2\2(2.2-h)l+ (62.4 x 0.8)[(2.2)'z(ft)]


lrFv = oI 3P = 162.4 x 2.221 (3.08 - 1'.4h + 0.8h)
BFpl+ 3P*=147
tytl
10,459.81 x + 769.1(0.28 - x) = 425 = (62.4 x 1,.6)l(2.2)2 (7.1)l + (62. 4 x 0.7)l(2.2)2 (1,.1)l
9690.71.x = 209.652 Y1= [52.4x2.T1(2.53)
x = 0.02'L6 m
r = 21.6 mm
IBF =W
162.4 x 2.Tl Q.08 - 1..U1+ 0.8ft) = [62.4 x 2.Tl(253)
Therefore; the block will float with 21.6 mm below the mercury surface. 3.08 - 1.4trx + 0.8h = 1.75 + 0.77
h = O.917 ft

Problem 3 - 74
A cube 2.2 teet on an Problem 3 -75 (CE May L997)
edge has its lower half of s.g. ='0.8 A 100-mm diameter solid cylinder is 95 mm high and weighing 3.75 N is
s.B. = 1.5 and upper half immersed in a liquid (y = 8.175 kN/m) contained in a tall metal cylinder
of s.g. = 0.7. It rests in a having a diameter of 125 mm. Before immersion, the liquid was75 mm deep.
two-layer fluid, with At what level will the solid cylinder float?
lower s.g. = '1..4 and
upper s.g. = 0.8.
Determine the heighf /r
of the top of the cube
above the interface. See
Figure 33.

Figure 33
THREE t ,r t
l7 o ;IfiI;ilH.',[ Force on Surfaces
FLUID MECHI\NICS
& HYDRAULICS
FLUID MECHANICS
& HYDRAULICS
CHAPTER
Totat Hydrostatic Force on Surfaces I I I
Solution Problem 3 - 76
L25 mm a
A wooden beam of sp. gr. 0.54 is 150 mm by 150 mm and is hinged at A, as
shown in the Figure. At what angle 0 will the beam float in water?

Solution

E T
I
E 1m
E
6
N

125 mm a
(d/ Eerore tmmerston (b) After immersion

Solve for the draft D in figure (b):


[Bf = 64
f tVo =W Weight of beam, IzV= yu"". Vh."*
't
i (8,175) Vo = 3.75
Weight of beam, 1tJ = (9,810 x 0.5a)[(0.15)'?(5)]
Vo = 0.0004587 m3 = 458,716 mm3
Weight of beam, W= 705.32 kN
+(100)'?xD=458,7"16
Draft,D=58.4mm Buoyant force, BF = y*arer VD
Buoyant force, BF = 9,810[(0.15)2 r]
when the solid cylinder is immersed, the liquid in the tall cylinder rises Buoyant force, BF = 220.725x
due to volume of liquid displaced. Therefore, the volume of liquid [>Ma = 0l
displaced equals the total volume of real and imaginary liquid above the BF(S - 0.5r) cos 0 = W(2.5 cos 0)
original level 220.725x(5 - 0.5x) = 7 06'32(2.5)
= VD
Vaboreorig. lerel 5r-0.512=8
g2s)z(x) = 458,7.16 0.5x2_5x+8=0
T
.r = 37.38 mm - (-s) r
2(0.s)
From Figure b:
x=2m
1

A=75+37.38-58.4 sin0= =j- =113


y = 53.98 mm
5-x
0 = 19.47"
Therefore; the solid cylinder will float with its bottom sg.gz6mm above the
bottom of the hallow cylinder.
l7? CHAPTER THREE
t.i Total Hydrostatic Force on Surfaces FLUID MECHANICS CHAPTER THREE
& HYDRAULTCS rotar Hydrostatic F;;;;;;;;;.:,r l7 3
Problem 3 - 77 (CE Board May 2003)
Problem 3 - 78 (CE Board November 1993)
From the figure below, it is shown th?i-the gate is
1.0 m wide and is hinged
the bottom of the gate. Compute the followirg: A boat going from salt water (sp. g.. = 1.03) to fresh water (sp. gr. = 1.0) sinks
(a) the hydrostatic force in kN acting on the 7.82 cm and after burning 72,730 kg of coal rises up by 15.-24-cm. Find the
gate,
(&) the location of the center of presi=ure original displacement of the boat in sea water in kN.
of tlie gate from the hinge,
(c) the minimum vorume,of concrete (unit weight
= 23.6 kN/m{ needed Solution
keep the gate in closed position.

Figure (a) Figure (b)

Solution
F =yi A:
F = 19.62 kN
e.81(1)(2 x 1)
r
0.5 m
4-*, D+0.0762-0.1524
v= +Q)=0.567m
[>Ma = 0]
i_
I
*"_ ,v = D - 0.0752

Fxy=Tx2.S ioo
19.62(0.66n = 2.57
T = 5.232 kN 2m
l3 Figure (c)
From the FBD of the
,ir concrete block:
t. we have to assume that the boat have a constant cross-sectional area A below
rl
[IFv = 0l the water surface and use ]water = 1000 kg/mo.
1il1i,
T + 3P =1,a1
l In Figure (a):
BF = YruVror, = 9.8L %on.
W = yro* V.on. = 23,6 Vrrr*
B\=77
T"nVo=W
5.?32+ g.g1 %o,,. = 23.6 V,o,,, w= (1000 x 1.03)IA(D)I
%o^. = 0.3796 ms W=1030AD ) Eq. (1)
r I

ll rli i

CHAPTER THREE FLUIDMECHANICS FtUtD MECHANICS CHAPTERTHREE I AT


il ,ll Surfaces I I 2
II +
i

Total Hydrostatic Force on Suifaces & HYDRAULICS & HYDRAULICS Total Hydrostatic Force on

iil
.ji In Figure (b): Solution
[l'
BFz-W For any floating body; Buoyant force = Weight
ii 'YwVo=W
{lr
I
w= 1000[A(D+0.0762)) Solving for displacement in sea water:
li
1

w= 1,0004(D + 0.0762) ) Eq. (2) Yseawater Vot=W


1.
il (64) Vu= 24,000 x 2,240
In Figure (c): Vor = 840,000 ft3 ,L
Bh=t7t1-rrrUO
1000[A(D -0.0762))=w-72730 ) Eq. (3) Solving for displacement in fresh water:
Voz=W
Tfreshwater
(52 2) (V oz) = 24,000 x 2240
From Eq. (1) and Eq. (2):
Voz= 864,308.68 fF
w=w ' Figure (a)
1030AD = 1000A(D + 0.0762)
1030D =1.OO0D+76.2 Let h be the difference in the drafts in fresh & seawater:
D = 2.54rn (draft in sea water) Voz- Vu = Area x h
. 864,308.68 - 840,000
From Eq. (1) 32,000
w = 1,030A(2.54) h = 0.76 ft
W = 261,6.24
Draft in fresh water, D = 34 + 0.76 = 34.76 tt
From Eq. (3)
10004(2.54 - 0.0762) = 261.6.24 -72730
2463.8A' = 26L6.2A - 72730 Problem 3 - 80 (CE Board November 1995)
A = 477.23 m2
Consider an arbitrary shaped body with a submerged volume Vs and a
density p,, submerged in a fluid of density py. What is'the net vertical force on
Therefore:
the body due to hydrostatic forces?
w = 261,6.2(477.23)
W = 1,248,529 kg (9.81/1000)
Solution
W=12248kN
Fnet= TVs
yf=vxg
Problem 3 - 79 Fn"t = P/8 Vs
A ship having a displacement of 24,000 tons and a draft of 34 feet in ocean
'fresh
enters a harbor of water. If the horizontal section of the ship at the
waterline is 32,000 sq. ft, what depth of fresh water is required to float the Problem 3 - 81
ii
slrip? Assume that marine ton is 2,240 lb and that sea water and fresh water A spherical balloon, g m in diameter is filled with helium gas pressurized to
weight 64 pcf and 62.2pcf, respectively. 111 kPa at a temperature of 20uC, and anchored by a rope to the ground:
Neglectingthe dead weight of the balloon, determine the tension in the rope.
Use R = TIZm/"Kfor helium gas and fav= 11.76 N/mr.
ir

t -r, CHAPTER THREE FLUID MECHANICS FLUI!} MECHANICS CHAPTER THREE


li II O Total Hydrostatic Force on Surfaces & HYDRAULICS & HYDRAULICS Total Hydrostatic Force on Surfaces 177
I Solution W",= 45.62N (from Figure 3 - 1)
ili
p _ 111x103 BF = y"* Vo
' RT 21,2(273 + 20) BF = e810(1.025)[(0.05)rL]
BF = 25.138L
Yherium = 7.787 N/mt
Tbuloon =
A
lxMo = 0l
|n(9/2)3
W,,,(L.5 cos 0) - BFI(L/2) cos 0l = 0
Vbuloon = 381.7 m3 45.62(1.5) - 25.138L(L / 2) = 0
[IFy = 0] 12.57 L2 = 68.43
BF:W-T=O L=2.33m
BF = Tun Vb"loon sin9 = 2/L
I
BF = 11.76(38L7) = 4488.8 N. sin 0 = 2/2.33
IzV= yr"[r. Vbaroon
0 =59o
I W=1,.787 (381.4 = 582.1N
I
4488.8-682.7=T
T = 3806.7 N
;

ll
Problem 3'83
A right circular cone is 100 mm in diameter and 200 mm high and weighs 1..6
Problem 3 - 82 N in air. How much force is required to push the cone (vertex downward)
into a body of liquid having sp. gr. of 0.8, so that its base is exactly at the
The buoy in Figure 3 - t has 80 N of steel weight attached. The buoy has
surface? How much additional force is required to push the base 10 mm
lodged against,a rock 2 m deep. Compute the angle 0 with the horizontal at below the surface?
which the buoy will leary assuming the rock exerts no moment on the buoy.
Solution
Solution
The required downward vertical force is:
F=BF-W
BF = ]tquto %one
6p = (9,810 x 0.8) [(n/3)(0.1,/2)2(0.2)l
BF = 4.11 N
F=4.11.-1..6
'F=2.51 N

Note: This force F= 2.51 N becomes t"


constant no rnatter how deep further
the cone is submerged.
I

t
I

I
I

I
I

t
"-]
I
ilmi, rHREE
rli
rl
t,
l7 8 ;Ifilff.:Ht[ Force on surraces
FLUID
&
FLUID MECHANICS
& HYDRAULICS
CHAPTER
Total Hydrostatic Force on Surfaces l79
t' '
rl
I
Problem 3 - 84
lr F
To what depth will a 2-m-diameter log, 4 m long and of sp. gr. 0.425 sink F=40N = 100N

I
t]" fresh water.
rl
Solution
For a homogeneous solid body floating on a
homogeneous liquid:

v, =
jlEL yo"o..
sliquid

arr=off at
As = 0.425nP (shaded area)
Let V = volume of wood
From geometery: In water:
As = Aru.tor - At i".gru [>Fv = 0]
0.425n* = Yz i 0,. - lz P stn 0 BFI-W-F=0
0,-sine=2.67 9810Y-W = 40
40+W
Solve 0 by trial and error:
,, -
9810
) Eq. (r)
Try 0 = 170"
ln glycerin:
770'@/180") - sin170" = 2.76 (*2.67)
[xFv = 0)
Try 0 = 1660 BFz-W-F=0
766"(n/1,80') - sin165' = 2.655 (*2.67) (9,810 x 1.3)Y- W=100
Try 0 =166.M" (e.810 x r.arlao*
w-l - r= roo
't66.44"(n/L80') - srn166.44'
= 2.67 O.K.
'L e81o .J

52+ 13W - W= 100


h=r_A W=1"60N
h = L - (.1) cos (0 /2)
h = 1. - (1) cos (1.66.M" /2) From Eq. (1):
ft = 0.882 m ,,_
v--
40+160
9810
7 = 0.0204 m3
Problem 3 - 85 _w160
unitweisht,Y= _=_
A block of wood requires a force of 40 N to keep it immersed in water and a v 0.0204
force of 100 N-to keep-it immersed in glycerin (rp. gr. = 1.3). Find the weight Unit weight, T = 7843 N/m3
and sp. gr. of the wood. Ywood
te'sr..
So. 5 = -7843
ywater 9810
Sp. gr.,s = 0.8

t
I 80 ;#|I;I,H:[ Force on Surraces
FLUID MECHANICS
& HYDRAULICS .
FLUID MECHANICS
& HYDRAULICS
CHAPTER THREE
Total Hydrostatic Force on Surfaces l8l
Problem 3 - 86 Since the volun\e of oil remain unchanged;
A rectangular tank of internal width of Vo,t (iniu"D = Vou (rinat)
5 m, as shown, contains oil of sp. gr. = (0.5XsX1.25) = (0.sxsx,x') - 0.1,274
0.8 and water. (a) Find the depth of oil, h'=L.30Lm
h. (b) lI a 1000-N block of wood is
As shown in Figure &, if the oi-l-water interface drops by a distance of y, the
floated in the oil, what is the rise in free
free surface of water will rise by y / 2, since the cross-sectional area of the
surface of the water in contact with air?
right compartment is twice that of the left compartment.

Sum-up pressure head from oil surface to water surface in m of water:


0 + 1.301(0.8) + (3 - y) - 4 - y/2= o
Solution 1.0408 -1-3y/2= 0
3y/z= 0.0408
y/2= 0.0136 m or 13.6 mm
Therefore; the free surface of water will rise 13.6 mm.

Problem 3 - 87
An open cylindrical tank 350 mm in diameter and 1.8 m high is inserted
vertically into a body of water with the open end down and floats with a 1300
N block of concrete (rp, gf. = 2.4) suspended at its lower end. Neglecting the
weight of the cylinder, to what depth will the open end be submerged in
water?
(a) I rsurtr (b)
Figure \u,,
Figure
Solution

(a) Depth of oil: (Refer to Figure a) [EFv = 0]


BF.o.. * BFol-w-a ) Eq. (1)
sum-up pressure head frorn oil surface o to water surface €) in m of water:
BF.on. = Ywater Vconc
lL+tr(o.B)+z-4= b I t Waona
Yy v conc -
T conc
0+0.8/,-1=0
h='1,.25m tf,.on.__ 1300
Di@
(b) Rise of the water surface: (Refer to Figure D)
V.on = 0.0552 m3
BF =W BF.onc = 9810(0.0552)
TotVo=W BF.on. = 541.7 N
(9810 x 0.8) Vo = 1000
Vo = 0.'1.274m3 BF.yr = Ywater VD
BF.yr = 9810[f; (0.35)'?h]
BF,y1=943.83h
__J
|
32 fl:iffil:,:ff Force on Surraces
FLUID M
&
FTUID MECHANICS
& HYDRAULICS Totat Hydrostatic
CHAPTER THREE
Force;il;;, I 83
Frorn Eq. (1)
541.7 + 943.83tt- 1300 = 0 Solution
/r = 0.803 m

Applying Boyles Law (taking p,t,. = 101.325


kpa) Wr = 205 kg
Before insertion:
Absolute pressure in a:u, pr = 101.325
kpa
Volunre of air insicle the cylinder, V., -------6-
= I(0.35)r(0.1S) 2.1 m v
Volume of air inside the cylinder, Vt
= 0.0173 mg 't,

After insertion:
1.8m
1
D
Absolute pressure in air, pz = 101.325 + yh H
Absolute pressure in ab, p2= 101.325 +
i.Af (O.SO3)
Absolute pressure in alu, p2= 109.2 kpa
Volume of air inside the cylinder, Vr=
Volunre of air insicle the cylinder,
t(O.eS;z
Vz = O.Og62x
* ',,1 t*,
[p-t V., = p, Vr) Figure a: High Tide
L01,.325 (0.1,73)
Figure b: Low T'ide
= 109.,2(0.0s 6Lx)
x=7.67m Weightof chain =12kg/m
x-h+!/= 1.g Density of steel = 7,790 kS/ *
y =1.8-1.67+g.gg3 Volume of steel (chain) = 12/7790
y = 0.933 m Volume of steel (chain) = 0.00154 m3 per meter length
Therefore' the open end is submerged
0.933 m berow the water surface.
In Figure a:
[>Fv = o]
Problem 3 - 8g (CE Board) Bfi+BPr-Wr-Wz=0 ri

A cylindrical buoy 600 rnrn in diameter and BF1 = y5s Ve


{
1..g m high weigrrs 205 kg.
moored i. salt water to uJ2.r\ te.gth It is = (1000 x 1.03)[f (0.6F(0re6)] i
of chai, weigf;ing iti.;;, m of its
length' At high tide, the height:f bd BFt = 2Z9.S8kg
0.84. Whar could be *." f,-rotruding above water surface is
of prot ,irtir., of the f,uoy if the ;;" dropped
2.1 m"? Density of steel is !ryfi
BFz = yr* V.r"in
7:7d0kg7m:. Ur" a""rity of waier
= 1000 kg/mt. = (1000 x 1.03)[0.00154(L)]
lr BF2= 15gU'
Wz=12L
279.58+ 1.586L - 205 - 12L = 0
L=7]1.5m

Depth of water, H = L + 0.9G


Depth of water, H = 8.12 m
--

l8+ fi,?iI;ilH:[ Force on surraces


FTUID MECHANICS
& HYDRAULICS
CHAPTERTHREE
Total Hydrostatic Force on
I QE
Surfaces I CDf
In Figure b: Solution
Depth of water, H' = H - 2.1.
Depth of water, H' = 6.02m Weight of ball:
Draft,D=H'-L' W= yuorr Vu"rr

Draft,D=6.02-L' w= (9810 x 0.42){n(0.15)e


W= 58.25 N
[rFy = 0]
BF'1+ BP'r-Wt -W'r= g Buoyant Force:
BFi = (tooo x 1.03))tf (0.6)rDl BF = y*ot", Vu"tt

BF'' = 291'U (6'02 - L') Br= (e810) f r(0.15)3


BF' t = 1753'18 - 291"23L' BF = 1,38.69 N
BF'2 = (1000 x 1.03)[0.00154(L' )) Depth of pool:
BF'1. = 1'5gU'' Work done by W= Work done by BF
w(4.3+h)=BF(h)
W'z= 12L'
58.25(4.3 +h)=138.69h
1753.18 - 291,23L', + 1,.586L', - 205 - 12L', = 0 h = 3.11m
L'= 5.13 m

D=6.02-5.13=0.89
Problem 3 - 90
y=1'8-D A hydrometer weighs 0.021.4 N and has a stem at the upper end which is Z.T9
y=1.8-0.89 mm in diameter. How much deeper will it float in oil (sp. gr. = 0.78) that in
y = 0.9't m (length of protrusiott)
alcohol (sp. gr. = 0.821)?

Solution
Problem 3 - 89 (CE Board)
In alcohol:
A wooden spherical ball with specific gravity of 0.42 and a diameter of 300 BF =W
mm is dropped from a height of 4.3 m above the surface of water in a pool of (9810 x 0.821)Vo,= 0.0214
unknown depth. The ball barely touched the bottom of the pool before it Voo= 2.657 x L0-6 m3
began to float. Determine the depth of the pool. Ven=/$$/ rr'6nt
InOil:
BF -W
(9810 x 0.78)V o, = 0.02'1.4
Von= 2.797 x L0-6 m3
Vpo= /,/S/ rrsP
i LVp=Vp6-Vpn
Alcohol, s = 0.821 Oil, s = 0.78
I .LVo=2,797 -2,657 = 140 mm3
LVo= f,(2.z97zlr=140
h=229mn

Ir
=-
l1
I

CHAPTER THREE FLUID MECHANICS CHAPTER THREE


I
I UO rotal Hydrostatic Force on surfaces
FLUID MECHANICS
& & HYDRAULICS Totat Hydrostatic Force on Surfaces 187

Problem 3 - 91 Problem 3 - 93
il
A plastic cube of side L and sp. gr.0.82 is placed vertically in water. Is A block of wood (sp. gr. = 0.64) is in the shape of a rectangular parallelepiped
having a 10-cm square base. If the block floats in salt water with its square
I

cube stable?
I base horizontal, what is its maximum height for stable equilibrium in the
Solution upright position?
The body is stable if Mis above G.
Sf3t Solution
Draft, D=
Note: The body is stable when M is 10cmx10cm
Draft, D = 0.82L
above G and uhstable if M is below
G. With smaller value of H, the
-l- = +(LXL)3
'vD
MB"=
' ,[ , :'", metacenter M will become higher

till
(L x t)(0.821)
than G making it rnuch stable.
MB,,= 0.102L When H increases, M will move
down closer to G making it less
GB"= L/2-D/2 I i-",.- l stable. Hence, the maximum

l
Seawater, s = 1,03
Gn,=6'9" height for stable equilibrium is
Since MB, > GB,,M is above G.
The body is stable. 'riil
waterliJe Seaion
when M coincides with G, or MBo=
GBo

From the figure:


Problem 3 - 92 GB"=H/2-D/2
A solid wood cylinder of specific gtavity 0.6 is 600 mm in diameter and Draft,D = ffiH=0.621H
mm high. If placed vertically in oil (sp. gr. = 0.85), would it be stable? GBo= 9.5, -0.621H/2
GB"=g'19"
Solution
sP'gr'wood, I
Draft,D- lMB"= *)
sp. gr. oil r=300mm vD

Draft, D = ffi trzoo) = 847 mm


MBo=
MB.= J-
(10)(10)D
VD
MBo=
100 _ 13.479
f (3oo)4 12(0.621,H) H
MBo=
n(3oo)2 (s47)
MB,=25.5U*^ oR: MB^= B' [, * tu"22 e) where o = oo
12D[ )
GBo = 5,gg -lz$ 7)
GB"=176.5 MB,,= 1o' (1+ o) = 13'419
Since MB, < GBo, the metacenter is below G.
12(0.62'tH\' H
Therefore, the body is unstable.
t 88 flfi;;H:[ff Force on surraces
FLUID MECHANICS
& HYDRAULICS
FLL'ID MECHANICS
& HYDRAULICS
CHAPTER
Total Hydrostatic Force on
rHREE gg
Surfaces I' v /
lMB,= GB")
Initial metacentric height, MG = MBo - GBo
13.479
= 0.189H lnibial metacentric height , MG = 77.49 - "1"17.45
lnitial metacenh'ic height, MG: -39.96 mm
H = 8.43 cm

Problem 3 - 95
Problem 3 - 94
A rectangular scow 9 m wide, 15 m long, and 3.6 m high has a draft in sea
A wood cone, 700 rnm diameter and 1,000 mm high floats in water with its water of 2.4 m. Its center of gravity is 2.7 m above the bottom of the scow.
vertex down. If the specific gravity of the wood ii o.oo, would it be stable?
Determine the following: .
Deternrine also its initial metacentric height.
(a) The initial metacentric height,
(b) The righting or overturning moment when the scow tilts until one side
Solution
is just at the point of submergence.
V*ood = $ n(350),(1000) K-- 350
I

V*ood = 128,287,700 ms i<- x- Solution


(n) Initial metacentric height:
Vp = Y V*o.a 70001a = 2s0 Metacenter, M
MB-= !-1, * tan2 e'l
'12D1
I Vo = 0.6 V*ooa
2l L=15m

Vo = 0.6 (128,281,700)
where 0 = 0o
Vo = 76,969,020 mm3

"
MB^= P)' [, * tu"'o'l
By similar solids: 12(2.4)L 2 l
vwoo,t _r1000)3
- l. D J MB, = 2'91"
v, ^
GB,= 2.7 - 1.2 = 1..5 m
B=9m

vu,oo,l _ lrooo13
osv-* -t D,/ Initial metacentric height, MG = MB, - GB"
Initial metacentric heighf MG = 2.8125 - 1..5
D:843.4 mm Initial metacentric height, MG = 1.3125 m
x _350 t
843.4 1000
Waterline Section
x = 295.2mm tano = ffi -
= "14.93
Q -*
MB^= I +Q95.2\4 ' 2.zn ,j J.3r{9- 1,2 n
'vD = sl
l,
ffi=77.4emm ' E-i
MB^=
't2Dln 2 l
tan2 i--}
I
From the Figure:
MB-= ez Il-1+'n.z/4.5\21
' '
K/ I
FI
:*./
3.6 rn

GB,=750 -3D/4
GB,=750 -3(843.4)/4
" 12(2.4)1 2 l I

GBo=117.nU*r, MB,=2.91
^
Since MBo < GBo, M is below G and the cone is UNSTABLE.

Anda mungkin juga menyukai